You are on page 1of 89

NBME Form 1 Step 2 Section 1:-1.

A 70-year-old woman has had increasing abdominal pain over the past 2 days. She has renal failure and has been receiving peritoneal dialysis for 18 months; her last treatment was 2 hours ago. She appears toxic. Her temperature is 39 C (102.2 F), and blood pressure is 140/90 mm Hg. Her abdomen is distended and diffusely tender to deep palpation with rebound tenderness. Leukocyte count is 18,000/mm3. Which of the following is the most appropriate next step? A ) X-ray films of the abdomen B ) Comparison of abdominal fluid amylase with serum amylase activity C ) Gram's stain of abdominal fluid D ) Ultrasonography of the abdomen E ) CT scan of the abdomen and pelvis 2. A 5-year-old girl is brought to the physician because of temperatures to 40 C (104 F), tachypnea, and a nonproductive cough for 12 hours. Four days ago she was treated with an oral antibiotic for suspected pneumococcal pneumonia. Examination shows diminished breath sounds over the lower right lung fields and dullness to percussion at the right costophrenic angle. Which of the following is the most likely diagnosis? A ) Bronchopleural fistula B ) Empyema C ) Lung abscess D ) Pleurodynia E ) Pneumothorax 3. A 40-year-old man is brought to the emergency department 1 hour after a high-speed motor vehicle collision. On arrival, he is awake and alert but has severe pain over the sternum. His systolic blood pressure is 80 mm Hg, pulse is 80/min, and respirations are 10/min. An ECG shows multifocal premature ventricular contractions but no ST-segment changes. His PO2 is 100 mm Hg. After 1 L of lactated Ringer's solution is administered, his PO2 decreases to 60 mm Hg while breathing 4 L/min of oxygen by nasal cannula. Pulmonary capillary wedge pressure has increased from 14 mm Hg to 24 mm Hg (N=110). Which of the following is the most likely explanation for the patient's poor response to fluid resuscitation? A ) Inadequate administration of fluids B ) Myocardial contusion C ) Myocardial infarction D ) Pulmonary contusion E ) Traumatic rupture of the aorta 4. A 21-year-old African American college student has had increasing fatigue over the past 3 weeks. Since an episode of cystitis treated with trimethoprim-sulfamethoxazole 3 weeks ago, he has been unable to keep up with his physical education classes. For 6 months, he has been following a vegetarian diet that has been supervised by student health services. Examination shows no abnormalities. His hemoglobin level is 10 g/dL, mean corpuscular volume is 85 m3, and reticulocyte count is 15%. Which of the following is the most likely cause of anemia in this patient?

A ) Anemia secondary to infection B ) Antibiotic therapy C ) Gastrointestinal blood loss D ) Sickle cell disease E ) Vegetarian diet 5. An 87-year-old nursing home resident with dementia, Alzheimer's type, is admitted to the hospital because of progressive lethargy and decreased appetite for 3 days. She had a flu-like illness followed by a deep cough 1 week ago. Over the past 10 months, she has been hospitalized once for bacterial pneumonia. She is responsive only to painful stimuli. Her temperature is 38.6 C (101.5 F), blood pressure is 110/60 mm Hg, pulse is 123/min and regular, and respirations are 28/min. Examination shows dry mucous membranes. There is no adenopathy. Crackles are heard in the right lung base. An x-ray film of the chest shows an infiltrate at the right lung base. The remainder of the examination shows no abnormalities. Which of the following is the most likely predisposing factor for this patient's pneumonia? A ) Decreased airway elasticity B ) Decreased baroreflex C ) Decreased gag reflex D ) Decreased thyroid function E ) Diastolic cardiac dysfunction F ) Impaired cardiac response to exercise G ) Impaired T-lymphocyte function H ) Impaired thirst I ) Increased lung compliance J ) Renal salt wasting 6. A 27-year-old woman comes to the physician because of feelings of anxiety about attending her 10-year high school reunion. She has a 2-year history of profound anxiety, palpitations, and sweating associated with an uneasiness around people; she avoids family gatherings and visiting friends because she is afraid of being embarrassed. She acknowledges that this fear is unreasonable. She does not use illicit drugs but says that alcohol makes her more comfortable around people. Her blood pressure is 130/90 mm Hg, and pulse is 88/min. On physical examination, she appears healthy and well nourished. Occasional wheezing is heard over the left lung field. The remainder of the examination shows no abnormalities. On mental status examination, she appears worried. Her leukocyte count is 9000/mm3 with a normal differential. Which of the following is the most likely diagnosis? A ) Alcohol abuse B ) Anxiety disorder due to a general medical condition C ) Asthma D ) Generalized anxiety disorder E ) Panic disorder with agoraphobia

F ) Social phobia The response options for the next two items are the same. You will be required to select one answer for each item in the set. For each patient with urinary incontinence, select the most likely cause. A ) Detrusor instability B ) Interstitial cystitis C ) Overflow incontinence D ) Stress incontinence E ) Urethra diverticulum F ) Urinary fistula 7. A previously healthy 44-year-old woman, gravida 4, para 4, comes to the physician because of a 9-month history of progressive loss of small amounts of urine while running; she now has to wear an absorbent pad. Examination shows a second-degree cystourethrocele. For each patient with urinary incontinence, select the most likely cause. A ) Detrusor instability B ) Interstitial cystitis C ) Overflow incontinence D ) Stress incontinence E ) Urethra diverticulum F ) Urinary fistula 8. One day after an uncomplicated spontaneous vaginal delivery, a 23-year-old woman, gravida 1, para 1, has the onset of loss of small amounts of urine. She received epidural anesthesia during labor and delivery. Examination shows an episiotomy without evidence of hematoma. She is voiding 50 to 75 mL of urine at a time. Postvoid residual volume is 300 mL. The response options for the next two items are the same. You will be required to select one answer for each item in the set. For each patient with cognitive impairment, select the most likely diagnosis. A ) Acute stress disorder B ) Dementia, Alzheimer's type C ) Dissociative amnesia D ) General paresis E ) Head trauma F ) Hepatolenticular degeneration (Wilson's disease)

G ) HIV encephalitis H ) Huntington's disease I ) Major depressive disorder J ) Multi-infarct (vascular) dementia K ) Niacin deficiency L ) Normal-pressure hydrocephalus M ) Parkinson's disease N ) Pick's disease O ) Schizophrenia, catatonic type P ) Normal aging 9. An 82-year-old woman is brought to the physician by her granddaughter because of a 6-week history of increasing forgetfulness. She is a retired schoolteacher and lives independently. Her granddaughter is concerned because on several occasions she has left the stove on when she went to bed. During conversations with her granddaughter, she has difficulty remembering past events and seems unconcerned about her memory lapses. The patient describes trouble sleeping through the night and has had a decreased appetite resulting in a 4.5-kg (10-lb) weight loss over the past month. She has a history of similar symptoms 2 and 5 years ago that were successfully treated with medication. She appears unkempt and has poor personal hygiene. Her temperature is 37 C (98.6 F), blood pressure is 110/70 mm Hg, and pulse is 80/min and regular. Mental status examination shows psychomotor retardation, a flat affect, impaired ability to recall past events, and trouble repeating three numbers in sequence. She is unable to recall the names of recent presidents. Her serum urea nitrogen (BUN) level is 25 mg/dL, and serum creatinine level is 1.7 mg/dL. For each patient with cognitive impairment, select the most likely diagnosis. A ) Acute stress disorder B ) Dementia, Alzheimer's type C ) Dissociative amnesia D ) General paresis E ) Head trauma F ) Hepatolenticular degeneration (Wilson's disease) G ) HIV encephalitis H ) Huntington's disease I ) Major depressive disorder J ) Multi-infarct (vascular) dementia K ) Niacin deficiency L ) Normal-pressure hydrocephalus M ) Parkinson's disease

N ) Pick's disease O ) Schizophrenia, catatonic type P ) Normal aging 10. A 42-year-old computer science professor is brought to the physician by her husband, who reports insidious changes in his wife's personality and behavior. He reports that she believes that aliens have been speaking to her and tampering with their heating and air-conditioning systems. He says that she was upset when she turned 40 years old, and her symptoms have developed since that time. She was adopted, and her family history is unknown. Physical examination shows vermicular movements of the tongue and bilateral writhing motions of the upper extremities. Mental status examination shows indifference to her condition and mild to moderate difficulty with memory and calculations. 11. A 5-month-old boy is brought for a follow-up examination. He was born at 37 weeks' gestation and has had persistent wheezing since shortly after birth despite treatment with nebulized and oral bronchodilators and oral corticosteroids. His diet consists of 32 ounces of iron-fortified cow's milkbased formula daily. He appears well nourished and happy. On examination, there is moderate relief of wheezing with extension of the neck. Which of the following is the most likely mechanism of this infant's wheezing? A ) Allergic reaction to cow's milk B ) Aspiration of a foreign body C ) Compression of the airway by a vascular ring D ) Concurrent upper respiratory tract infection E ) Persistent immaturity of lungs 12. An 18-month-old boy is brought to the emergency department because he has not used his left arm since he fell while walking and holding hands with his 8-year-old sister 2 hours ago. On examination, he holds his left upper extremity at his side with his forearm pronated. There is no tenderness of the left lower extremity, but there is restricted movement of the elbow. The remainder of the examination shows no abnormalities. Which of the following is the most appropriate initial step in management? A ) Passive hypersupination of the forearm B ) Application of figure-of-8 strap C ) Administration of analgesics and application of ice D ) Aspiration of the elbow joint E ) In-place splint immobilization of the elbow 13. A 6-month-old girl is brought to the physician for a routine health maintenance examination. She was born with a lumbosacral myelomeningocele which was successfully repaired at 2 days of age. The anterior fontanelle is 6 x 8 cm and bulging, and the posterior fontanelle is 3 x 4 cm and bulging. She has severe motor and sensory deficits involving both lower extremities. A head growth chart shows the following values: Age Head circumference (cm) Birth 34.2 1 month 36.6 2 months

38 4 months 44 6 months 47 Which of the following is the most likely cause of increased intracranial pressure? A ) Acute cerebral edema B ) Decreased absorption of cerebrospinal fluid C ) Dilation of cerebral arteries D ) Intracranial mass lesion E ) Obstruction of lateral sinus F ) Obstruction of superior vena cava G ) Obstruction of cerebrospinal fluid flow H ) Overproduction of cerebrospinal fluid 14. An 18-year-old primigravid woman comes for her initial prenatal visit at 16 weeks' gestation. She is not sure about the date of her last menstrual period but says that the pregnancy probably occurred immediately after she stopped taking oral contraceptives 5 months ago. Maternal serum fetoprotein (MSAFP) level is increased to 3 multiples of the median. Which of the following is the most appropriate next step in management? A ) Repeat measurement of MSAFP level B ) Triple screening for MSAFP, serum -hCG, and serum estriol levels C ) Ultrasonography D ) Amniocentesis for measurement of -fetoprotein level E ) Amniocentesis for chromosomal analysis 15. A 28-year-old man is brought to the emergency department by police because of severe pain in the right lower quadrant of the abdomen for 36 hours. He has been incarcerated in the county jail, and his court hearing is scheduled in 12 hours. He takes no medications. Abdominal examination shows no rebound. Complete blood count, liver tests, and erythrocyte sedimentation rate are within normal limits. An x-ray film of the abdomen shows a normal gas pattern. Test of the stool for occult blood is negative. Which of the following is the most likely diagnosis? A ) Adjustment disorder B ) Conversion disorder C ) Depressive disorder not otherwise specified D ) Factitious disorder E ) Hypochondriasis F ) Malingering G ) Somatization disorder 16. A 6-year-old girl is brought to the physician because of a 4-week history of headache, fatigue,

and decreased appetite. During this period, she has had nausea and vomiting. At the age of 4 years, she was diagnosed with poststreptococcal glomerulonephritis. She is at the 15th percentile for height and the 10th percentile for weight. Examination shows no abnormalities. Her serum urea nitrogen (BUN) level is 50 mg/dL. Which of the following is most likely to limit progression of this patient's renal failure? A ) Increased potassium and sodium bicarbonate intake B ) Decreased sodium and daily calorie intake C ) Low-protein diet D ) Strict fluid restriction E ) Dialysis 17. A previously healthy 62-year-old man comes to the emergency department because of abdominal pain for 48 hours. His temperature is 38.6 C (101.5 F), blood pressure is 130/80 mm Hg, pulse is 110/min, and respirations are 15/min. Abdominal examination shows diffuse left lower quadrant tenderness with no peritoneal signs. Rectal examination shows no abnormalities; test of the stool for occult blood is negative. His leukocyte count is 14,700/mm3. Which of the following is the most appropriate next step in diagnosis? A ) Barium enema B ) CT scan of the abdomen C ) Colonoscopy D ) Cystoscopy E ) Exploratory laparotomy 18. A previously healthy 3-month-old girl is brought to the emergency department because of a 3day history of grunting and increasing difficulty breathing. She appears ill. Her temperature is 36.7 C (98 F), pulse is 160/min, and respirations are 76/min. Examination shows grunting, nasal flaring, and marked intercostal retractions. Bronchial breath sounds and occasional bilateral crackles are heard on auscultation. Serum studies show: Ca2+ 5.6 mg/dL Phosphorus 11 mg/dL Alkaline phosphatase 250 U/L Capillary blood gas analysis on 100% oxygen: pH 7.36 PCO2 38 mm Hg PO2 46 mm Hg An x-ray film of the chest shows bilateral, diffuse interstitial infiltrates and absence of the thymic shadow. Bronchoalveolar lavage is positive for numerous Pneumocystis carinii. Which of the following is the most likely mechanism of these findings?

A ) Adenosine deaminase deficiency B ) Consumption of complement C ) Defective opsonization D ) Destruction of CD4+ T lymphocytes E ) Developmental arrest of maturation of B lymphocytes F ) Dysmorphogenesis of the third and fourth pharyngeal pouches G ) Impaired chemotaxis H ) Impaired phagocytic oxidative metabolism 19. A 4-year-old boy is brought to the emergency department 20 minutes after being involved in a motor vehicle collision. He was an unrestrained passenger. On arrival, his blood pressure is 110/70 mm Hg, pulse is 100/min, and respirations are 32/min with grunting and retractions. Examination shows multiple bruises over the chest. Arterial blood gas analysis while breathing 40% oxygen shows: pH 7.38 PCO2 34 mm Hg PO2 66 mm Hg An x-ray film of the chest obtained 4 hours later shows diffuse infiltrates on the right side. Which of the following is the most likely diagnosis? A ) Acute respiratory distress syndrome B ) Aspiration pneumonia C ) Fat embolism D ) Hemothorax E ) Pulmonary contusion 20. A 72-year-old man comes to the physician because of a 7-month history of leg weakness and dry eyes and mouth. He also has had a 10.4-kg (23-lb) weight loss over the past 4 months despite no change in appetite. He has smoked one and a half packs of cigarettes daily for 50 years. He drinks 4 oz of alcohol daily. He has peptic ulcer disease and emphysema. Medications include cimetidine, theophylline, and low-dose prednisone. Examination shows mild ptosis. He has a barrelshaped chest. Breath sounds are distant. There is moderate weakness of the proximal muscles of the lower extremities. Reflexes are absent. He has difficulty rising from a chair. Sensory examination shows no abnormalities. An x-ray film shows a hyperinflated chest and a 3 x 4-cm mass in the right hilum. His neurologic findings are most likely due to a lesion involving which of the following? A ) Muscle membrane B ) Parasympathetic nervous system C

) Peripheral nerve D ) Presynaptic neuromuscular junction E ) Sympathetic nervous system 21. A 42-year-old man comes to the emergency department because of a 2-week history of increasingly severe headaches and a 2-day history of nausea, vomiting, neck stiffness, and unsteadiness. He has type 2 diabetes mellitus treated with glyburide. His temperature is 38.1 C (100.5 F). Funduscopic examination shows bilateral papilledema. Neurologic examination shows mild meningismus and diffusely brisk deep tendon reflexes. He walks with a moderately broadbased gait. He is able to recall two out of three objects after 5 minutes and makes several errors on serial sevens. A CT scan of the head shows no abnormalities. Cerebrospinal fluid analysis shows a glucose level of 18 mg/dL, a protein level of 108 mg/dL, and a leukocyte count of 59/mm3 (1% segmented neutrophils and 99% lymphocytes); a cryptococcal antigen assay is positive. Which of the following is the most appropriate pharmacotherapy for this patient? A ) Acyclovir B ) Amphotericin B C ) Itraconazole D ) Penicillin E ) Vancomycin 22. A 3-year-old girl is brought to the physician after her mother noted blood on her underpants. Examination shows genital condylomata acuminata in the perineal, peri-introital, labial, and anal areas. Some of the pedunculated condylomata appear to have caused the bleeding. She has no visible intravaginal condylomata or vaginal or anal tears. Her mother has a palmar wart on her hand but no history of condylomata acuminata. Her mother has a boyfriend who does not live with them and who has never been left alone with the girl. They live with the mother's 27-year-old brother who only baby-sits the children when they are asleep. Which of the following is the most appropriate next step in management? A ) Psychiatric assessment of the mother B ) DNA typing of the mother's palmar wart for papillomavirus C ) Treatment of the mother's palmar wart D ) Vaginal, anal, and throat cultures for Chlamydia trachomatis and Neisseria gonorrhoeae in the child E ) Laser therapy of the condylomata acuminata in the child 23. A 55-year-old man has had crushing substernal chest pain on exertion over the past 6 weeks. He had a myocardial infarction 2 months ago. He takes nitroglycerin as needed and one aspirin daily. He has smoked two packs of cigarettes daily for 30 years. Examination shows normal heart

sounds and no carotid or femoral bruits. Treatment with a -adrenergic blocking agent is most likely to improve his symptoms due to which of the following mechanisms? A ) Decreasing diastolic relaxation B ) Decreasing myocardial contractility C ) Dilating the coronary arteries D ) Peripheral vasodilation E ) Preventing fibrin and platelet plugs 24. A previously healthy 52-year-old woman comes to the physician because she has had a large pimple on her right hand for 2 weeks that has failed to heal. She resides in southeastern USA where she owns a nursery and garden shop. Examination shows a painless red papule on the hand with several nontender subcutaneous nodular lesions above it. Which of the following is the most likely diagnosis? A ) Blastomycosis B ) Candidiasis C ) Coccidioidomycosis D ) Histoplasmosis E ) Sporotrichosis 25. A 28-year-old woman at 28 weeks' gestation reports excessive fatigability and dyspnea. Her blood pressure is 118/74 mm Hg, pulse is 110/min and regular, and lungs are clear to auscultation. The cardiac apex is not palpable. S1 is loud, and there is a sharp sound after S2. A low-frequency diastolic murmur is heard at the apex that increases in intensity before S1. Which of the following is the most likely diagnosis? A ) Aortic regurgitation B ) Ebstein's anomaly C ) Mitral regurgitation D ) Mitral stenosis E ) Tricuspid regurgitation 26. A 27-year-old woman comes to the physician because of a 2-year history of intermittent diarrhea and severe cramping abdominal pain. The stools are watery, occasionally foul-smelling, and nonbloody. She is currently pain-free and has not had diarrhea for 2 days. She also has intermittent constipation. She has not had fever or weight loss. She returned from a trip to Mexico 3 months ago. She had an appendectomy at the age of 12 years and a cesarean delivery 4 years ago. Examination shows no abnormalities. Which of the following is the most likely diagnosis?

A ) Bacterial gastroenteritis B ) Crohn's disease C ) Intermittent small-bowel obstruction D ) Irritable bowel syndrome E ) Laxative abuse 27. An asymptomatic 21-year-old woman is found to have an adnexal mass on pelvic examination. She uses oral contraceptives. A photograph of the mass is shown. Which of the following is the most likely diagnosis? A ) Benign cystic teratoma B ) Corpus luteum cyst C ) Dysgerminoma D ) Endometrioma E ) Mucinous cystoadenoma 28. A 16-year-old girl is brought to the physician because of episodes of palpitations over the past 6 months. The episodes occur when she runs or plays basketball. She is otherwise asymptomatic. Her blood pressure is 124/46 mm Hg, pulse is 78/min, and respirations are 18/min. She weighs 55 kg (121 lb) and is 180 cm (71 in) tall. Her arm span is 188 cm (74 in), and the upper segment to lower segment ratio is 0.85. Her fingers appear long and are hyperextensible. A grade 4/6, early diastolic murmur is heard along the upper and middle left sternal border with radiation to the apex. Peripheral pulses are bounding. Which of the following is the most likely cause of these findings? A ) Aortic incompetence B ) Aortic stenosis C ) Mitral incompetence D ) Mitral stenosis E ) Pulmonary incompetence F ) Pulmonary stenosis G ) Tricuspid incompetence

H ) Tricuspid stenosis 29. A previously healthy 87-year-old woman comes to the physician because of a 4-month history of vulvar itching. Examination shows excoriated areas from scratching and a white, thin vulva. The labia minora are absent, and there are small fissures at the introitus. The remainder of the examination shows no abnormalities. Which of the following is the most likely diagnosis? A ) Escherichia coli infection B ) Lichen sclerosus C ) Squamous cell carcinoma D ) Trichomoniasis E ) Vulvar melanoma F ) Vulvar vestibulitis 30. One month after undergoing an uneventful renal transplant for chronic renal failure secondary to glomerulonephritis, a 38-year-old woman is hospitalized because of increased serum urea nitrogen (BUN) and creatinine levels. Prior to transplantation, she had been receiving hemodialysis for 3 years. Current medications include cyclosporine and prednisone. Examination shows no abnormalities. Over the past 48 hours, urine output has remained stable. Both renal biopsy and a radionuclide scan confirm the diagnosis of acute rejection. Which of the following is the most effective treatment? A ) Immediate discontinuation of cyclosporine B ) Increased dosage of corticosteroids C ) Diuresis and alkalinization of the urine D ) Renal dialysis for 12 weeks E ) Transplant nephrectomy 31. An 18-year-old man comes to the physician 1 week after he had a blood pressure of 140/110 mm Hg during a routine precollege examination. His temperature is 37.1 C (98.7 F), blood pressure is 140/100 mm Hg, pulse is 92/min, and respirations are 12/min. The upper extremities appear to be more muscular than the lower extremities. Radial pulses are normal; femoral, posterior tibial, and dorsalis pedis pulses are decreased. A grade 2/6 systolic murmur is heard over the precordium, anterior chest, and back. An ECG shows left ventricular hypertrophy. Which of the following is the most appropriate next step in management? A ) Limiting physical activity B ) Repeat blood pressure measurement in 1 month

C ) Initiate a low-sodium diet and exercise program D ) Pharmacologic management E ) Operative treatment 32. A 77-year-old woman comes to the physician because of a 2-day history of cramping abdominal pain and distention accompanied by nausea and vomiting. She is otherwise healthy and has no history of abdominal operations. Her temperature is 37.4 C (99.4 F), blood pressure is 110/86 mm Hg, pulse is 112/min, and respirations are 24/min. Cardiopulmonary examination shows no abnormalities. Examination of the abdomen shows distention and mild diffuse tenderness; bowel sounds are high-pitched. An x-ray film of the abdomen shows air-fluid levels throughout the small bowel and air in the liver; there is no gas in the colon or free air. Which of the following is the most likely diagnosis? A ) Adhesive small-bowel obstruction B ) Cecal cancer C ) Gallstone ileus D ) Intussusception E ) Mesenteric infarction F ) Ruptured appendicitis G ) Small bowel lymphoma 33. Four hours after undergoing a cesarean delivery at term followed by tubal ligation, a 37-year-old woman, gravida 2, para 2, has dizziness and confusion. The operation was uncomplicated, and blood loss is estimated to be 800 mL. Patient-controlled epidural analgesia has been moderately effective for pain. Her blood pressure now is 80/40 mm Hg, decreased from 120/72 mm Hg intraoperatively, and pulse is 152/min, increased from 96/min intraoperatively. Breath sounds are decreased bilaterally. No murmurs are heard. Abdominal examination shows distention and tenderness. Bowel sounds are absent. The incision is intact with no drainage. She is disoriented to person, place, and time. Her hematocrit is 23%; preoperative hematocrit was 35%. Which of the following is the most likely cause of the hemodynamic changes? A ) Epidural-related hypotension B ) Insufficient intraoperative fluid replacement C ) Postoperative intra-abdominal hemorrhage D ) Supine hypotensive syndrome E ) Underestimated intraoperative blood loss 34. A 42-year-old woman comes to the physician for evaluation of persistently increased blood

pressures. At her last two office visits during the past 3 months, her blood pressure has ranged between 150170/105115 mm Hg. During this period, she has had occasional headaches. In addition, she has had an increased urine output over the past 6 weeks that she attributes to a diet high in sodium. She is otherwise healthy and takes no medications. Her blood pressure today is 168/115 mm Hg, pulse is 68/min, and respirations are 14/min. Funduscopic examination shows mild arteriovenous nicking. The point of maximal impulse is not displaced. There is no edema, abdominal bruits, or masses. Serum studies show: Na+ 144 mEq/L Cl 90 mEq/L K+ 2.9 mEq/L HCO3 32 mEq/L Urea nitrogen (BUN) 20 mg/dL Creatinine 1.2 mg/dL Which of the following is the most likely underlying cause of this patient's hypertension? A ) Autonomous production of aldosterone B ) Catecholamine-producing tumor C ) Decreased arterial distensibility caused by atherosclerosis D ) Excess production of atrial natriuretic peptide E ) Juxtaglomerular cell hypertrophy and sclerosis 35. A previously healthy 4-year-old girl is brought to the physician because of fever and refusal to walk for 1 day. She appears mildly ill. Her temperature is 38.6 C (101.5 F), pulse is 120/min, and respirations are 22/min. The right knee is erythematous and swollen. She holds her right knee in flexion and resists any attempted movement of her right leg. She cries when the right knee is moved. Which of the following is the most appropriate next step in management? A ) Acetaminophen with codeine therapy B ) Arthrocentesis C ) Bone marrow aspiration D ) Bone scan E ) Immobilization and traction F ) Lyme titer

G ) MRI of the spine H ) Physical therapy I ) Reassurance J ) Serum rheumatoid factor assay K ) Systemic antibiotic therapy 36. An 18-year-old man comes for an examination prior to participation in school sports. He states that he has had a dull ache in the scrotum since being hit in that area during a basketball game 2 months ago. Examination shows a 2-cm, hard, nontender mass in the right testicle. The mass does not transilluminate or change in size when the patient is placed in the supine position. Which of the following is the most likely cause? A ) Cystic dilations of the efferent ductules B ) Dilated pampiniform venous plexus C ) Fluid accumulation within the tunica vaginalis testis D ) Germinal cell tumor E ) Vascular trauma 37. A 14-year-old boy is brought to the physician by his parents because of a 2-year history of increasing academic problems. His parents say that he has always been hyperactive and distractible, but now his academic performance has deteriorated to the point that he is failing ninth grade. His teachers say that his hyperactivity is disrupting the classroom. He weighs 54 kg (120 lb) and is 152 cm (60 in) tall. Sexual development is Tanner stage 5; examination shows macro-orchidism, which was not shown on previous examinations. He has a high forehead and long, protruding ears. He exhibits poor eye contact during the examination. Psychoeducational testing shows an IQ of 70. Which of the following is the most likely diagnosis? A ) Attention-deficit/hyperactivity disorder B ) Autistic disorder C ) Down syndrome D ) Fetal alcohol syndrome E ) Fragile X syndrome F ) Lesch-Nyhan syndrome

G ) Pervasive developmental disorder, not otherwise specified H ) Prader-Willi syndrome I ) Rett's disorder J ) Seminiferous tubule dysgenesis (Klinefelter's syndrome) 38. A 32-year-old woman comes to the physician because of vaginal discharge for 2 weeks. She has been sexually active with one female partner for 5 years. She has not been treated with antibiotics over the past 2 years. Her last Pap smear was 6 years ago when she was sexually active with a male partner. She has not used illicit drugs or alcohol. Examination shows a grayish vaginal discharge with a pH greater than 4.5. A wet mount preparation of the vaginal discharge is most likely to show which of the following? A ) Budding yeast B ) Clue cells C ) Ferning D ) Leukocytes in sheets E ) Trichomonas vaginalis 39. A 57-year-old woman with breast cancer comes to the physician because of increasing neck pain over the past 3 days. She has fallen frequently because of muscle weakness. Vital signs are within normal limits. Examination shows hyperreflexia of all extremities. There is tenderness over the cervical spine. Serum calcium level is 11 mg/dL. X-ray films show metastases to the cervical spine. Which of the following is the most appropriate next step in management? A ) Application of a soft cervical collar B ) Physical therapy C ) Mithramycin therapy D ) Tamoxifen therapy E ) Spinal cord decompression and cervical stabilization 40. A 57-year-old man comes to the physician because of intermittent urinary incontinence over the past 6 months. He has loss of small amounts of urine when he coughs or sneezes. He has not had pain or blood with urination. He has a 15-year history of type 2 diabetes mellitus with peripheral neuropathy, retinopathy, and gastroparesis. Current medications include metoclopramide and glyburide. He appears well. Rectal examination shows a normal-sized prostate. Neurologic examination shows decreased sensation in a stocking-glove distribution. Achilles tendon reflexes are

absent bilaterally. Test of the stool for occult blood is negative. Urinalysis shows 2+ protein with no leukocytes or erythrocytes. His postvoid residual volume is 500 mL. Which of the following is the most likely mechanism of this patient's incontinence? A ) Central nervous system disorder B ) Functional incontinence C ) Intrinsic sphincter deficiency D ) Overflow incontinence from acontractile bladder E ) Overflow incontinence from bladder outlet obstruction F ) Pelvic floor muscle weakness G ) Retroperitoneal fibrosis H ) Retroperitoneal lymphadenopathy I ) Urinary tract infection 41. A 27-year-old primigravid woman at 38 weeks' gestation is admitted in labor. Her pregnancy has been uncomplicated, and a routine prenatal visit 2 days ago showed no abnormalities. On admission, fetal heart tones cannot be heard. Ultrasonography shows little amniotic fluid, fetal edema, and no evidence of a fetal heartbeat. After 1 hour, she delivers a 3175-g (7-lb) stillborn infant; examination of the infant shows no obvious abnormalities except for mild edema. The placenta and membranes appear normal. Which of the following is the most appropriate immediate course of action? A ) Notify the hospital liability department B ) Obtain consent for fetal organ donation from the parents C ) Recommend autopsy of the infant D ) Tell the mother not to worry since she can get pregnant again E ) Tell the parents that there is a 1 in 4 chance of recurrence in future pregnancies 42. A previously healthy 16-year-old high school wrestler comes to the physician because of a rash on his forearms and the back of his legs for 1 week. He is allergic to pollen and dust. Examination shows patches of erythema with mild lichenification over the antecubital and popliteal fossae. There are clusters of painful umbilicated vesicles at sites of active skin inflammation. Which of the following is the most likely diagnosis? A ) Eczema herpeticum

B ) Herpes zoster C ) Keratosis pilaris D ) Lichen planus E ) Pityriasis rosea 43. A 5-week-old boy is brought to the physician because of vomiting for 3 days. Switching from a cow's milk-based formula to a soy-based formula and one bottle of an electrolyte solution has not decreased his vomiting. His mother says that there is no yellow color to the vomitus, but it is forceful and occurs immediately after he has had 1 to 2 ounces of liquid. He appears to vomit more liquid than he drank. He has one mustard-colored seedy stool daily. Examination shows no abnormalities. Which of the following is the most likely explanation for his vomiting? A ) Duodenal atresia B ) Gastroesophageal reflux C ) Hypertrophic pyloric stenosis D ) Lactose intolerance E ) Protein malabsorption F ) Rotavirus infection 44. A 28-year-old woman is hospitalized after taking a massive overdose of acetaminophen tablets in a suicide attempt. She has type 1 diabetes mellitus and major depressive disorder refractory to tricyclic antidepressant therapy. Despite appropriate therapy, she develops rapidly progressive hepatic failure and becomes progressively encephalopathic. On the 6th day of hospitalization, she is comatose. A CT scan of the brain shows mild diffuse swelling. An appropriately crossmatched, sizeappropriate donor liver is available. Which of the following is the most appropriate course of action regarding transplantation? A ) Do not proceed with the transplantation because diabetes mellitus is a contraindication B ) Do not proceed with the transplantation because hepatic function is likely to return over the next week C ) Do not proceed with the transplantation because major depressive disorder places the patient at risk for another suicide attempt D ) Do not proceed with the transplantation because the onset of encephalopathy and CT findings suggest bacterial meningitis E

) Proceed with the transplantation 45. A 3-year-old boy who is HIV positive is brought for a routine examination. His diet is appropriate for age. His medications include three antiretroviral drugs and trimethoprim-sulfamethoxazole for Pneumocystis carinii prophylaxis. Laboratory studies show: Hemoglobin 8.6 g/dL Mean corpuscular hemoglobin 38 pg/cell Mean corpuscular hemoglobin concentration 30% Hb/cell Mean corpuscular volume 101 m3 Leukocyte count 5600/mm3 Segmented neutrophils 60% (many hypersegmented) Bands 3% Lymphocytes 37% Red cell distribution width 21% (N=1016) Which of the following is most likely to have prevented this patient's anemia? A ) Folic acid supplementation B ) Iron supplementation C ) Thyroid supplementation D ) Vitamin B12 (cyanocobalamin) supplementation E ) Monthly intravenous immune globulin therapy 46. A 20-year-old man is brought to the emergency department on a summer day 20 minutes after developing headache, nausea, and unsteady gait while running the last 2 miles of a marathon. On arrival, he is confused and disoriented. His temperature is 40 C (104 F), blood pressure is 100/60 mm Hg, and pulse is 155/min. His skin is warm and dry. Neurologic examination shows no focal findings. Which of the following is the most likely mechanism of this patient's condition? A ) Depletion of total body potassium B ) Depletion of total body sodium C ) High-output cardiac failure D ) Inadequate dissipation of body heat E ) Release of creatine kinase from muscle cells _______________________________________________________________________________ ___ Form 1--Section 2:-1. A 77-year-old woman is brought to the physician by her son for a routine health maintenance examination. She says that she feels well. Her son reports that 1 month ago, she got lost while

driving home from the local supermarket. Two weeks ago, she forgot to turn off the stove after cooking dinner. She has been wearing bilateral hearing aids since audiometry 2 years ago showed bilateral high-frequency hearing loss. Her visual acuity corrected with glasses is 20/25 in both eyes. Neurologic examination shows mild fine tremors of the hands when the arms are outstretched; the tremor is not present at rest. Muscle strength is 5/5 in all extremities. Deep tendon reflexes are decreased at the ankles and 2+ elsewhere. Her gait is normal. Sensation to vibration is mildly decreased over the toes. On mental status examination, she is awake, alert, and conversant. Her language function is normal. She is oriented to person, place, and time and recalls one out of three objects after 10 minutes. Which of the following findings in this patient warrants further evaluation? A ) Decreased deep tendon reflexes at the ankles B ) Decreased sensation to vibration over the toes C ) High-frequency hearing loss D ) Memory loss E ) Tremor of the outstretched hands 2. A 10-year-old girl is brought to the emergency department because of diffuse, aching abdominal pain, nausea, and recurrent vomiting over the past 5 hours. She has an 8-year history of type 1 diabetes mellitus treated with 20 U of NPH and 6 U of regular insulin in the morning and 14 U of NPH and 5 U of regular insulin in the evening. She appears lethargic but is easily arousable. There is an obvious odor of ketones on her breath. Her blood pressure is 100/70 mm Hg, pulse is 95/min, and respirations are 20/min and deep. Serum studies show: Na+ 142 mEq/L K+ 5.3 mEq/L HCO3 6 mEq/L Glucose 710 mg/dL Which of the following laboratory findings is most likely to be increased? A ) Arterial pH B ) Serum C-peptide level C ) Serum magnesium level D ) Serum osmolality E ) Serum phosphorus level 3. A healthy 24-year-old woman comes for a routine health maintenance examination. Menses occur at regular 28-day intervals and last 5 to 6 days. Her last menstrual period was 3 weeks ago. She takes no medications. Bimanual examination shows a 5-cm, mildly tender left adnexa. A pregnancy test is negative. Which of the following is the most appropriate next step in management? A ) Repeat examination in 2 weeks B ) Measurement of serum CA 125 level C ) Measurement of serum -fetoprotein level

D ) CT scan of the pelvis E ) Diagnostic laparoscopy 4. A 24-year-old primigravid woman at 18 weeks' gestation comes for a routine prenatal visit. She has had increased bowel movements over the past 9 weeks; the stools are sometimes covered with mucus and blood. Use of over-the-counter antidiarrheal drugs has not relieved her symptoms. Pregnancy has been otherwise uncomplicated. She has never traveled outside the USA. Examination shows erythematous, tender nodules over the anterior surface of both lower extremities; some of the nodules have a violaceous hue. The uterus is consistent in size with an 18-week gestation. Rectal examination shows no hemorrhoids or fissures. Fetal heart tones are audible by Doppler. Which of the following is the most likely diagnosis? A ) Amebiasis B ) Diverticulitis C ) Hyperperistaltic diarrhea D ) Inflammatory bowel disease E ) Viral gastroenteritis 5. A 32-year-old woman at 38 weeks' gestation comes for a routine prenatal visit. During routine screening at 28 weeks' gestation, she tested positive for hepatitis B surface antigen. Her pregnancy has been otherwise uncomplicated. Examination shows a uterus consistent in size with a 38-week gestation. Which of the following measures is most likely to decrease the risk for hepatitis B infection in her newborn? A ) Recommendation of bottle-feeding rather than breast-feeding B ) Maternal administration of hepatitis B immune globulin (HBIG) now C ) Neonatal administration of HBIG after delivery and hepatitis B vaccine at 3 months of age D ) Neonatal administration of HBIG and hepatitis B vaccine immediately after delivery E ) Cesarean delivery 6. A 37-year-old woman comes to the physician because of progressive shortness of breath over the past 5 years; she now has fatigue and shortness of breath with mild exertion. She has a history of mitral stenosis secondary to rheumatic fever at the age of 15 years. She was asymptomatic until 5 years ago when she developed severe shortness of breath during pregnancy. She was treated with diuretics, low-sodium diet, and bed rest, and she was able to deliver the baby at term. Her only medication is hydrochlorothiazide. Her temperature is 37 C (98.6 F), blood pressure is 110/80 mm Hg, pulse is 100/min and regular, and respirations are 26/min. Cardiac examination shows an obvious opening snap in S2. A grade 3/6, late diastolic murmur is heard at the apex. A right ventricular lift is palpated along the left sternal border. Which of the following is most likely increased in this patient?

A ) Blood flow to the lower lung fields B ) Diastolic filling time C ) Left-to-right shunt of blood D ) Left ventricular end-diastolic pressure E ) Pulmonary artery pressure 7. A 5-year-old girl with ventricular septal defect is scheduled for tonsillectomy in 2 weeks. She has no known drug allergies. Her temperature is 37 C (98.6 F). Examination shows no abnormalities. Which of the following is the most appropriate prophylaxis prior to tonsillectomy? A ) Amoxicillin B ) Ciprofloxacin C ) Rifampin D ) Tetracycline E ) Trimethoprim-sulfamethoxazole F ) No prophylaxis indicated 8. An 8-year-old girl with type 1 diabetes mellitus is brought to the emergency department 10 minutes after being involved in a motor vehicle collision. She was in the back seat of a small automobile that was rear-ended. Initially, she was alert during transport and reported bilateral thigh pain, but then she stopped talking, closed her eyes, and became unresponsive to voice; on arrival, she responds to noxious stimuli with brief grimaces and no withdrawal. Her blood pressure is 40/palpable mm Hg, pulse is 148/min, and respirations are 28/min. Air entry is symmetric. The pupils are equal and react to light. No cardiac murmur is heard. The abdomen is soft. There is swelling of the upper portions of both thighs. Her hematocrit is 37%. Which of the following is the most appropriate next step in management? A ) Measurement of arterial blood gases B ) X-ray film of the chest C ) CT scan of the head D ) Administration of 50% dextrose in water E ) Infusion of 0.9% saline

9. A previously healthy 16-year-old boy is brought to the emergency department 20 minutes after an episode of left arm shaking that lasted approximately 3 minutes. Over the past 2 days, he has had fever and emotional lability. On arrival, his temperature is 38.9 C (102 F). He is somnolent and disoriented to person, place, and time. He responds poorly to pain. Neurologic examination shows no other abnormalities. Laboratory studies show: Hematocrit 34% Leukocyte count 6000/mm3 Segmented neutrophils 50% Lymphocytes 50% Platelet count 280,000/mm3

Analysis of cerebrospinal fluid shows: Leukocyte count 120/mm3 Segmented neutrophils 20% Lymphocytes 80% Erythrocyte count 300/mm3 Glucose 60 mg/dL Protein 400 mg/dL Which of the following is the most likely cause of this patient's neurologic findings? A ) Bacterial infection B ) Congenital malformation C ) Fungal infection D ) Hemorrhage E ) Immune-mediated demyelination F ) Parasitic infection G ) Viral infection 10. A 72-year-old man with hypertension has had increasingly severe back pain over the past 2 months. He had a myocardial infarction 4 years ago. He has marked tenderness over T11, T12, L1, and L2. An x-ray film of the lumbosacral spine shows osteoblastic lesions in these vertebrae. Which of the following is the most likely diagnosis? A ) Abdominal aneurysm B ) Fibrosarcoma C ) Metastatic prostate carcinoma

D ) Multiple myeloma E ) Osteosarcoma 11. A 64-year-old woman has moderately severe postoperative pain 1 day after a total abdominal hysterectomy and bilateral salpingo-oophorectomy. Which of the following is the most appropriate analgesic pharmacotherapy? A ) Oral aspirin-codeine compound B ) Oral diazepam C ) Oral ibuprofen D ) Intermittent intravenous naloxone E ) Patient-controlled intravenous morphine F ) Transcutaneous administration of fentanyl 12. A 37-year-old woman comes to the physician because of a 1-day history of throbbing facial pain. She describes the pain as 7 out of 10 in intensity. Over the past 9 days, she has had nasal congestion, purulent nasal discharge, sore throat, and a nonproductive cough. She does not smoke. Her husband and children have had no recent illness. Her temperature is 38.5 C (101.3 F). Examination shows congested nasal mucosa and purulent discharge on the left. There is tenderness to palpation over the left cheek and no transillumination over the left maxillary sinus. The tympanic membranes are normal, and there is no erythema of the throat. Examination shows no cervical adenopathy. The lungs are clear to auscultation. Which of the following is the most likely causal organism? A ) Haemophilus influenzae type b B ) Moraxella catarrhalis C ) Staphylococcus aureus D ) Streptococcus pneumoniae E ) Streptococcus pyogenes (group A) 13. A 42-year-old woman comes to the physician for an annual pelvic examination and Pap smear. Over the past year, she has had increasing fatigue and difficulty sleeping. She has two children who both attend college. She is currently looking for part-time work outside the home. Her husband has been busy in a new start-up business. Examination shows no abnormalities. Laboratory studies show: Hemoglobin 15 g/dL Mean corpuscular volume 95 m3 Leukocyte count 6000/mm3 with a normal differential

Serum Na+ 145 mEq/L Cl 102 mEq/L K+ 4.5 mEq/L HCO3 25 mEq/L Urea nitrogen (BUN) 18 mg/dL Creatinine 1.0 mg/dL Alkaline phosphatase 70 U/L Aspartate aminotransferase (AST, GOT) 22 U/L Alanine aminotransferase (ALT, GPT) 19 U/L -Glutamyltransferase (GGT) 83 U/L (N=550 U/L) Which of the following is the most likely explanation for this patient's laboratory abnormalities? A ) Acetaminophen B ) Alcohol C ) Diphenhydramine D ) Estrogen effect E ) Ibuprofen 14. Five weeks after vaginal delivery of a healthy full-term newborn, a 22-year-old woman, gravida 1, para 1, is brought to the physician by her mother because of depressed mood for 2 weeks. Her mother is concerned that her daughter is not able to take care of her infant. Physical examination shows no abnormalities. She is quiet and tearful and does not engage in conversation easily. She states that she lives alone with her infant and has had thoughts of suicide and infanticide. Which of the following is the most appropriate next step in management? A ) Reassurance B ) Long-term outpatient counseling C ) Antipsychotic therapy D ) Selective serotonin reuptake inhibitor therapy E ) Admission to the hospital for treatment 15. A 72-year-old man comes for a routine follow-up examination. He has chronic obstructive pulmonary disease treated with -adrenergic agonists and ipratropium by metered-dose inhaler and mild arterial insufficiency of the lower extremities treated with aspirin. His blood pressure is 160/60 mm Hg, pulse is 70/min, and respirations are 12/min. Funduscopic examination shows arteriovenous nicking. Pedal pulses are decreased bilaterally. Which of the following antihypertensive drugs is most likely to cause adverse effects in this patient?

A ) 2-Adrenergic agonist B ) -Adrenergic blocking agent C ) -Adrenergic blocking agent D ) Angiotensin-converting enzyme (ACE) inhibitor E ) Calcium-channel blocking agent F ) Loop diuretic G ) Thiazide diuretic H ) Vasodilator The response options for the next two items are the same. You will be required to select one answer for each item in the set. For each patient with loss of consciousness, select the most likely diagnosis. A ) Aortic stenosis B ) Carotid sinus hypersensitivity C ) Conversion reaction D ) Hypertrophic obstructive cardiomyopathy E ) Hypoglycemia F ) Mitral valve prolapse G ) Orthostatic hypotension H ) Pulmonary embolus I ) Seizure J ) Vasovagal syncope K ) Vertebrobasilar insufficiency

16. A 15-year-old boy is brought to the emergency department 30 minutes after a 2-minute episode of loss of consciousness after completing a 400-meter race. On awakening, he says that he feels fine except for shortness of breath. He weighs 82 kg (180 lb) and is 191 cm (75 in) tall. His blood pressure is 110/70 mm Hg, pulse is 70/min and regular, and respirations are 15/min. The lungs are clear to auscultation. A grade 2/6 systolic murmur is heard at the left sternal border with minimal radiation to the neck; the murmur becomes louder when he stands. For each patient with loss of consciousness, select the most likely diagnosis. A ) Aortic stenosis B ) Carotid sinus hypersensitivity C ) Conversion reaction D ) Hypertrophic obstructive cardiomyopathy E ) Hypoglycemia F ) Mitral valve prolapse G ) Orthostatic hypotension H ) Pulmonary embolus I ) Seizure J ) Vasovagal syncope K ) Vertebrobasilar insufficiency 17. A 62-year-old woman is brought to the emergency department 1 hour after a 1-minute episode of loss of consciousness; her symptoms began when she stood up after she passed a dark, watery stool. She has had diarrhea and dark stools for 2 days. She has been receiving warfarin therapy for deep venous thrombosis for 2 weeks. On arrival, her blood pressure is 82/60 mm Hg, and pulse is 150/min and regular. She is unable to stand. The lungs are clear to auscultation. A grade 2/6 systolic murmur is heard at the second right intercostal space with no radiation. Examination shows a soft, nontender abdomen. There is 1+ edema of the right lower extremity with no tenderness. Test of the stool for occult blood is positive. 18. A 50-year-old woman has had progressive dyspnea over the past 2 weeks and constant, sharp chest pain for 4 days. The pain is localized to the center of the chest and is worse while supine. She underwent a right, modified radical mastectomy and adjuvant chemotherapy for breast cancer 3 years ago. She has a history of hypothyroidism treated with thyroid replacement therapy. She has smoked one pack of cigarettes daily for 30 years and drinks two ounces of alcohol daily. She is dyspneic and diaphoretic. Her temperature is 37.2 C (99 F), blood pressure is 90/70 mm Hg with a pulsus paradoxus of 20 mm Hg, pulse is 110/min, and respirations are 28/min. Examination shows jugular venous distention to the angle of the mandible. The liver span is 14 cm with 4 cm of shifting abdominal dullness. Arterial blood gas analysis on room air shows a pH of 7.50, PCO2 of 30 mm Hg,

and PO2 of 70 mm Hg. An x-ray film of the chest shows an enlarged cardiac silhouette with a globular configuration. An ECG shows sinus tachycardia with nonspecific ST-segment changes diffusely. Which of the following is the most appropriate next step in management? A ) Echocardiography B ) CT scan of the abdomen C ) Ventilation-perfusion lung scans D ) Bronchoscopy E ) Paracentesis 19. A 3-year-old boy is brought to the physician because of a 7-day history of fever and a painful swollen lymph node in his groin. This is his sixth episode of lymph node swelling; the previous episodes resolved after drainage and prolonged antibiotic therapy. He also had pneumonia at the age of 12 months that required chest tube placement for drainage. A maternal uncle died during childhood of recurrent infections. The patient is at the 5th percentile for height and weight. His temperature is 38.5 C (101.3 F). Examination shows a warm, tender, erythematous lymph node in the right inguinal area. There are several healed incisions over the inguinal area and neck from old drainage sites. Laboratory studies show: Hematocrit 35% Leukocyte count 17,000/mm3 Segmented neutrophils 65% Bands 10% Lymphocytes 25% Platelet count 350,000/mm3 A Gram's stain of the lymph node aspirate shows numerous segmented neutrophils filled with bacteria; cultures grow Staphylococcus aureus. Which of the following is the most likely mechanism for these findings? A ) Adenosine deaminase deficiency B ) Consumption of complement C ) Defective opsonization D ) Destruction of CD4+ T lymphocytes E ) Developmental arrest of maturation of B lymphocytes F ) Dysmorphogenesis of the third and fourth pharyngeal pouches G ) Impaired chemotaxis H ) Impaired phagocytic oxidative metabolism 20. A 67-year-old woman has been intubated for 1 week after undergoing a left lobectomy for lung cancer. She has chronic obstructive pulmonary disease. Her preoperative functional vital capacity was 40% of predicted. She is awake and alert. Her blood pressure is 130/75 mm Hg, and pulse is 72/min. The ventilator settings are a synchronized intermittent mandatory ventilation of 8/min,

FIO2 of 40%, and positive-end expiratory pressure of 5 cm H2O. Arterial blood gas analysis shows: pH 7.42 PCO2 47 mm Hg PO2 90 mm Hg O2 saturation 96% Which of the following is the most appropriate next step in management? A ) Antibiotic therapy B ) Bronchodilator therapy C ) Chest physiotherapy D ) Decrease inotropes E ) Diuretic therapy F ) Fiberoptic bronchoscopy G ) Heparin therapy H ) Incentive spirometry I ) Increase FIO2 J ) Increase inotropes K ) Increase respiratory rate L ) Placement of thoracostomy tube M ) Tracheostomy N ) Wean from the ventilator 21. A 67-year-old woman is brought to the emergency department because of severe chest pain 4 hours after undergoing outpatient endoscopy and dilatation of an esophageal stricture caused by reflux. At discharge, she reported no chest pain. Three hours later, she vomited a small amount of blood and had severe pain. She is pale. Her temperature is 38 C (100.4 F), blood pressure is 140/85 mm Hg, pulse is 125/min, and respirations are 22/min. Examination shows crepitus in the neck and moderate epigastric tenderness. The lungs are clear to auscultation, and breath sounds are equal bilaterally. Rectal examination shows no masses; test of the stool for occult blood is positive. Which of the following is the most likely cause of these symptoms? A ) Bleeding from erosive esophagitis

B ) Esophageal perforation C ) Mallory-Weiss syndrome D ) Myocardial infarction E ) Perforated gastric ulcer 22. An 87-year-old woman is brought to the physician by her son because of progressive memory loss over the past 2 years. Her son says that she repeats herself frequently and has been forgetting to take her routine medications. She takes hydrochlorothiazide for mild systolic hypertension and levothyroxine for hypothyroidism. She had vulvar cancer 10 years ago treated with wide excision. Her blood pressure is 138/78 mm Hg. Physical examination is within normal limits for her age. MiniMental State Examination score is 23/30. Laboratory studies, including serum vitamin B12 (cyanocobalamin), thyroxine (T4), and thyroid-stimulating hormone levels, are within normal limits. A CT scan of the head shows mild volume loss. Which of the following is the most appropriate pharmacotherapy? A ) -Adrenergic agonist B ) Cholinesterase inhibitor C ) Dopamine agonist D ) Prednisone E ) Selective serotonin reuptake inhibitor 23. A newborn is in severe respiratory distress immediately following delivery. She was born at 35 weeks' gestation to a 35-year-old woman, gravida 2, para 1, aborta 1, who did not receive prenatal care. The newborn's pulse is 60/min, and respirations are irregular and labored. Examination shows pallor with perioral cyanosis, anasarca, hepatosplenomegaly, and scattered petechiae. Cord blood hemoglobin is 4 g/dL, and reticulocyte count is 18%. A direct antiglobulin (Coombs') test is positive. Which of the following sets of blood groups is most likely in the mother and her newborn? Mother Newborn A ) A, Rh-positive O, Rh-positive B ) A, Rh-positive O, Rh-negative C ) A, Rh-negative O, Rh-negative

D ) O, Rh-positive O, Rh-negative E ) O, Rh-negative O, Rh-positive 24. After an uncomplicated laparoscopic cholecystectomy, a 62-year-old man has not had any urine output since the Foley catheter was removed 12 hours ago. During the hour before the operation, the 40 minutes of operating room time, and the 2 hours in the recovery room, his fluid input was 2.5 L and urine output was 1 L. Since that time, he has been receiving intravenous 5% dextrose in water with 0.45% saline and morphine. He is awake and alert and has a moderate amount of abdominal pain. Preoperative serum studies showed: Na+ 137 mEq/L K+ 4.2 mEq/L Urea nitrogen (BUN) 18 mg/dL Creatinine 1.2 mg/dL One hour after receiving an intravenous bolus of 0.9% saline, the patient does not produce any urine. Which of the following is the most appropriate next step in management? A ) Increase in the dose of morphine B ) Intravenous administration of an additional bolus of 0.9% saline C ) Intravenous administration of doxazosin D ) Intravenous administration of furosemide E ) Reinsertion of a Foley catheter 25. A healthy 55-year-old man comes for an initial health maintenance examination. His last visit to a physician was over 10 years ago. He does not smoke and drinks only on social occasions. Examination shows no abnormalities. Which of the following immunizations should be administered? A ) Hepatitis A vaccine B ) Influenza virus vaccine C ) Measles-mumps-rubella vaccine D ) Pneumococcal vaccine E ) Diphtheria-tetanus toxoid 26. A 19-year-old man comes to the physician because of frequent nosebleeds over the past 3 weeks. He has bipolar disorder currently well controlled with lithium carbonate, bupropion, and valproic acid. Physical examination shows no abnormalities except for dried blood in the nares. Mental status examination shows an anxious mood and slight motor restlessness. Serum studies show a lithium carbonate level of 1.3 mEq/L (therapeutic range=0.61.2), and valproic acid level of

77 g/mL (therapeutic range=40100). Which of the following is the most appropriate next step in management? A ) Measurement of serum aspartate aminotransferase (AST, GOT) activity B ) Measurement of serum bupropion level C ) Platelet count D ) Discontinuation of lithium carbonate therapy E ) Discontinuation of valproic acid therapy 27. An asymptomatic 32-year-old man comes for a routine health maintenance examination. He has a 10-year history of frequent sinus and pulmonary infections. He had an anaphylactic reaction to a blood transfusion following a motor vehicle collision 3 years ago. His temperature is 37 C (98.6 F). Examination shows mild erythema in the posterior pharynx. The lungs are clear to auscultation. A complete blood count and serum protein electrophoresis are within normal limits. Which of the following is the most likely cause of the frequent infections? A ) Colonization with Streptococcus pneumoniae B ) Common variable immunodeficiency C ) HIV infection D ) Selective IgA deficiency E ) X-linked agammaglobulinemia 28. A 37-year-old man is brought to the emergency department 6 hours after the onset of constant, increasingly severe abdominal pain and nausea. His symptoms awoke him from sleep, and he has vomited once since that time. He has no history of similar symptoms, and he does not take any medications or use alcohol or illicit drugs. Family history is noncontributory. He is in acute distress and lying in the fetal position. Any movement exacerbates the pain. His temperature is 37.8 C (100 F), blood pressure is 108/68 mm Hg, pulse is 112/min, and respirations are 24/min. The lungs are clear to percussion and auscultation. Examination shows a rigid abdomen; bowel sounds are absent. Laboratory studies show: Hemoglobin 14 g/dL Leukocyte count 18,200/mm3 Platelet count 150,000/mm3

Serum Urea nitrogen (BUN) 34 mg/dL Creatinine 1.9 mg/dL

Total bilirubin 1.2 mg/dL

An x-ray film of the chest shows a small amount of free air under the left diaphragm. Administration of antibiotics and fluids is begun. Which of the following is the most appropriate next step in management? A ) Barium swallow B ) CT scan of the abdomen C ) Intravenous administration of an H2-receptor blocking agent D ) Upper endoscopy E ) Laparotomy 29. A 52-year-old woman with alcoholism comes to the physician after a serum cholesterol level of 290 mg/dL was found on a routine screening. She drinks a pint of vodka daily. She takes captopril for hypertension and glyburide for type 2 diabetes mellitus. She also has intermittent episodes of gout. Fasting serum studies show: Total cholesterol 252 mg/dL HDL-cholesterol 80 mg/dL Triglycerides 300 mg/dL Glucose 118 mg/dL Thyroid-stimulating hormone 4.5 U/mL Which of the following is the most appropriate next step in management? A ) Alcohol cessation B ) Better control of diabetes C ) Switch from captopril to calcium-channel blocking agent therapy D ) Gemfibrozil therapy E ) Thyroid replacement therapy 30. A 23-year-old woman has pain, cramping, and swelling of the right calf 3 days after an uncomplicated labor and delivery. The right foot is swollen, and there is marked tenderness with dorsiflexion and palpation of the right calf. Examination shows no other abnormalities. A complete blood count and serum electrolyte levels are within normal limits. Which of the following is the most likely cause of this condition? A ) Hypercoagulable state of pregnancy B ) Hyperuricemia

C ) Peripheral artery aneurysm D ) Platelet embolus E ) Prolonged pressure on the vena cava during delivery 31. A 2-month-old boy is brought to the physician for a well-child examination. He smiles spontaneously and vocalizes without crying, but he does not appear to laugh or squeal. He will not work for a toy that is out of his reach. Which of the following is the most appropriate assessment of language and psychosocial development? Language Psychosocial development development A ) Normal normal B ) Normal delayed C ) Delayed normal D ) Delayed delayed 32. A previously healthy 24-year-old woman comes to the physician because of a low-grade fever and a nonproductive cough for 7 days. She has been able to continue her daily activities. Her temperature is 37.7 C (99.9 F). A few scattered inspiratory crackles are heard in the thorax. An xray film of the chest shows patchy infiltrates in both lungs. Which of the following is the most appropriate initial pharmacotherapy? A ) Amoxicillin B ) Cefaclor C ) Ciprofloxacin D ) Erythromycin E ) Trimethoprim-sulfamethoxazole 33. A 45-year-old woman comes to the emergency department because of shortness of breath,

chest pain, dizziness, and mild numbness and tingling around the lips for 2 hours. She says that she feels like she is going to die. She had three similar episodes last week when she was vacationing at the Grand Canyon; the first episode occurred while crossing a narrow bridge on a donkey. She takes a hypoglycemic drug for type 2 diabetes mellitus, verapamil for hypertension, and sumatriptan as needed for migraine. She is mildly diaphoretic and appears pale. Her blood pressure is 130/90 mm Hg, pulse is 120/min, and respirations are 28/min. Serum glucose level is 120 mg/dL. An ECG shows sinus tachycardia. Sublingual nitroglycerin therapy does not relieve her symptoms and gives her a headache. The most appropriate next step in management is administration of wh ich of the following? A ) Haloperidol B ) Lorazepam C ) Oxygen D ) Sumatriptan E ) Verapamil 34. A 67-year-old man is brought to the emergency department 4 hours after the onset of severe midlumbar back pain. He is anxious, pale, and diaphoretic. His temperature is 37.1 C (98.8 F), blood pressure is 105/65 mm Hg, and pulse is 120/min. Examination shows no other abnormalities. X-ray films of the lumbar spine show degenerative disc disease with calcifications anterior to the vertebral bodies. Which of the following is the most likely diagnosis? A ) Aortoiliac occlusion B ) Herniated nucleus pulposus C ) Lumbar discitis D ) Lumbar strain E ) Pyelonephritis F ) Ruptured aortic aneurysm G ) Spinal stenosis 35. A 17-year-old boy is brought to the emergency department by his parents because of bizarre behavior for 6 hours. Last night he was out with friends, and since returning, he has been confused and has "trashed" his room. His blood pressure is 165/95 mm Hg. He is hypervigilant, has little spontaneous speech, and is disoriented to place and time. He appears catatonic but abruptly becomes assaultive two times and needs to be restrained. Which of the following is the most likely substance taken? A ) Cocaine

B ) Ecstasy C ) LSD D ) Methaqualone E ) PCP The response options for the next two items are the same. You will be required to select one answer for each item in the set. For each patient with back pain, select the most likely diagnosis. A ) Herniated disc B ) Lumbar spinal stenosis C ) Metastatic cancer D ) Muscle strain E ) Osteoporotic compression fracture F ) Sacroiliitis G ) Spinal epidural abscess H ) Spondylolisthesis 36. A 57-year-old woman is brought to the physician 2 days after the sudden onset of severe low back pain; the pain does not radiate to the lower extremities. The pain began when she was lifting her grandson. She does not have weakness or sensory loss in the legs and has had no urinary incontinence. She has a 10-year history of rheumatoid arthritis treated with prednisone. Her temperature is 37 C (98.6 F), blood pressure is 130/60 mm Hg, and pulse is 64/min. Examination shows deformities of the interphalangeal joints of the hands and exquisite tenderness to percussion over the lumbar spine. Bilateral straight-leg raising to 80 degrees does not increase the pain. Muscle strength and sensation are intact in the lower extremities. Deep tendon reflexes are 2+ bilaterally. Babinski's sign is absent bilaterally. For each patient with back pain, select the most likely diagnosis. A ) Herniated disc B ) Lumbar spinal stenosis C

) Metastatic cancer D ) Muscle strain E ) Osteoporotic compression fracture F ) Sacroiliitis G ) Spinal epidural abscess H ) Spondylolisthesis 37. A previously healthy 32-year-old plumber comes to the physician because of a 3-week history of constant, dull, low back pain that does not radiate to the extremities. The pain began after he unloaded heavy equipment from his van. It increases with activity and is temporarily relieved by bed rest and ibuprofen. Examination shows tenderness to palpation over the lumbar paraspinal region bilaterally. The pain increases with forward or lateral movements of the spine. Muscle strength and sensation are intact in the lower extremities. Bilateral straight-leg raising to 80 degrees does not increase the pain. Deep tendon reflexes are 2+ bilaterally. Babinski's sign is absent bilaterally. 38. A 72-year-old man comes to the physician because of a 2-month history of urination twice nightly and occasional urinary frequency and urgency. He has a 15-year history of type 2 diabetes mellitus now moderately well controlled with glyburide. His father was diagnosed with prostate cancer at the age of 70 years, and his sister died of complications from systemic lupus erythematosus. His blood pressure is 135/86 mm Hg. Cardiopulmonary examination shows no abnormalities. Abdominal examination shows no suprapubic fullness or tenderness. There is mild enlargement of the prostate with no palpable nodules. His postvoid residual volume is 10 mL. Serum studies show a urea nitrogen (BUN) level of 45 mg/dL and creatinine level of 3.8 mg/dL. Urine dipstick shows 3+ protein. Which of the following is most likely to have prevented progression of this patient's renal disease? A ) Intermittent Foley catheterization B ) Intravenous mannitol therapy C ) Oral cyclophosphamide and prednisone therapy D ) Oral enalapril therapy E ) Oral finasteride therapy F ) Oral prednisone therapy only G ) Oral terazosin therapy 39. Two hours ago, a 24-year-old man had the sudden onset of pain in the right side of his chest that has become increasingly severe. He is now having difficulty breathing. His temperature, blood pressure, and pulse are normal. An x-ray film of the chest is shown. Which of the following is the most appropriate next step in management?

A ) Bed rest and sedative therapy B ) Antibiotic therapy C ) Anticoagulant therapy D ) Tube thoracostomy E ) Immediate thoracotomy 40. A 21-year-old college student comes to the physician because of acne that developed 4 days ago while she was taking her medical college admission test. She is concerned about her appearance and plans to be in a wedding in 3 weeks. She has had similar episodes that have resolved completely without treatment. Examination shows acute acne over the face with a predominance of comedones and pustules. There is no evidence of chronic scarring. Which of the following is the most appropriate initial step in treatment? A ) Dietary restriction of chocolates and simple sugars B ) Dietary restriction of milk products C ) Topical acyclovir D ) Topical hydrocortisone cream E ) Topical retinoic acid 41. A 67-year-old woman comes for a routine health maintenance examination. She exercises regularly. She is not sexually active. At her last visit 1 year ago, her serum cholesterol level was 180 mg/dL, and fasting serum glucose level was 80 mg/dL; a Pap smear and mammography showed normal findings. Two years ago, flexible sigmoidoscopy showed no abnormalities. Today, she weighs 63 kg (140 lb) and is 165 cm (65 in) tall. Her blood pressure is 120/80 mm Hg. Examination shows no abnormalities. Which of the following is the most appropriate screening test for this patient? A ) Pap smear B ) Measurement of serum cholesterol level C ) Measurement of serum glucose level D ) Mammography E ) Flexible sigmoidoscopy 42. A 62-year-old man has had the gradual onset of fatigue and shortness of breath over the past 3 years. There is striking jugular venous distention with a large wave occurring with S2. The carotid

upstroke is normal. Cardiac examination shows a lifting systolic motion of the sternum and no palpable point of maximal impulse. A grade 3/6, holosystolic, plateau-shaped murmur that is loudest on inspiration is heard at the lower left sternal border. The liver is enlarged and tender, and the abdomen is swollen with a fluid wave. There is marked ankle edema. Which of the following is the most likely cause of the murmur? A ) Aortic stenosis B ) Mitral regurgitation C ) Mitral stenosis D ) Tricuspid regurgitation E ) Ventricular septal defect 43. A 24-year-old woman comes to the physician because of constant, severe pain in her neck, shoulders, and back for 3 months. She has been unable to enjoy her usual activities because of the pain. Use of over-the-counter ibuprofen and aspirin has not relieved her symptoms. She has a history of irritable bowel syndrome. Examination shows multiple tender spots over the neck, shoulders, and lumbar spine. Range of motion of all joints is full. There is no evidence of synovitis. Fluorescent serum antinuclear antibody and rheumatoid factor assays are negative. Which of the following is the most likely diagnosis? A ) Ankylosing spondylitis B ) Fibromyalgia C ) Polymyalgia rheumatica D ) Polymyositis E ) Seronegative rheumatoid arthritis 44. A 72-year-old man comes to the physician because of a 6-month history of mild to moderate shortness of breath when climbing stairs. He had a myocardial infarction 2 years ago and has had an ejection fraction of 35% since then. His only medication is a -adrenergic blocking agent. The lungs are clear to auscultation. Cardiac examination shows an S4 gallop. There is no peripheral edema. Laboratory studies are within normal limits. An ECG shows no acute changes. Which of the following is the most appropriate pharmacotherapy? A ) -Adrenergic blocking agent B ) Angiotensin-converting enzyme (ACE) inhibitor C ) Angiotensin2-receptor blocking agent D ) Nitrates

E ) Thiazide diuretic 45. A 42-year-old woman comes to the physician because of a 3-month history of a recurrent vivid dream that several men are assaulting her and her children. Upon awakening, she is anxious and distressed by the frightening images. She realizes that it is just a dream but is afraid to go back to sleep. She does not know why she is having this particular dream because she has never been the victim of an assault. She drinks two to three cups of coffee each morning. She does not use illicit drugs. Physical examination shows no abnormalities. There is no evidence of depressed mood or hallucinations. Laboratory studies are within normal limits. Which of the following is the most likely diagnosis? A ) Acute stress disorder B ) Nightmare disorder C ) Panic disorder D ) Sleep apnea E ) Sleep terror disorder 46. A 57-year-old man is brought to the emergency department 30 minutes after he was found on the floor of his house; he has left hip pain and shortness of breath. He has renal failure but has missed his last two dialysis treatments. His renal failure was caused by inadvertent ingestion of ethylene glycol. His renal function did not improve, and he is currently on the transplantation list. Medications include amlodipine and doxazosin. On arrival, his temperature is 37.5 C (99.5 F), blood pressure is 150/100 mm Hg, pulse is 95/min and regular, and respirations are 24/min. His breathing is rapid and deep. Crackles are heard in the lung bases. Examination shows a soft abdomen. Bowel sounds are normal. The left lower extremity is externally rotated. Laboratory studies show: Serum Na+ 135 mEq/L Cl 102 mEq/L K+ 7.1 mEq/L HCO3 12 mEq/L Arterial blood gas analysis on 4 L/min of oxygen by nasal cannula: pH 7.22 PCO2 31 mm Hg PO2 61 mm Hg An ECG shows peaked T-waves. It will be at least 45 minutes before dialysis can be started. Which of the following is the most appropriate next step in management? A ) Observation until dialysis is initiated B ) Intravenous calcium gluconate C ) Intravenous glucose and insulin D

) Intravenous 0.9% saline E ) Intravenous sodium bicarbonate F ) Rectal sodium polystyrene sulfonate (Kayexalate) _______________________________________________________________________________ ___________________________________________________ Section 3:-1. A 26-year-old woman is brought to the emergency department because of marked confusion for 2 hours; she also has had a flu-like illness for 3 days. Over the past 6 weeks, she has had increased fatigue, weakness, and nausea. She recently started thyroid hormone replacement therapy for autoimmune thyroiditis; 1 week ago, her serum thyroid-stimulating hormone level was 3 U/mL. Her temperature is 38 C (100.4 F), blood pressure is 80/40 mm Hg, and pulse is 140/min. She appears confused and lethargic. Examination shows cool, mottled skin. There is generalized hyperpigmentation, especially involving the palmar creases. The lungs are clear to auscultation. Abdominal examination shows diffuse mild tenderness and no rebound. Laboratory studies show: Hemoglobin 10 g/dL Leukocyte count 9000/mm3 Segmented neutrophils 55% Eosinophils 20% Lymphocytes 25% Serum Na+ 124 mEq/L Cl 92 mEq/L K+ 6.4 mEq/L HCO3 16 mEq/L An x-ray film of the chest and urinalysis show normal findings. An ECG shows sinus tachycardia with peaked T waves. Which of the following is most likely to confirm the primary cause of this patient's condition? A ) Measurement of pulmonary artery pressure B ) Measurement of right atrial pressure C ) Measurement of serum antithyroglobulin antibody level D ) Measurement of serum lactate dehydrogenase activity E ) Measurement of serum thyroid-stimulating hormone level F ) ACTH stimulation test G ) Dexamethasone suppression test H ) Blood cultures I ) Echocardiography

2. A 27-year-old man comes to the physician because of a 1-week history of shortness of breath with exertion, paroxysmal nocturnal dyspnea, and swelling of his feet. He has not had chest pain or palpitations. He has been healthy except for a "bad cold" 1 month ago that resolved spontaneously after 10 days. His temperature is 37 C (98.6 F), blood pressure is 90/60 mm Hg, pulse is 120/min, and respirations are 24/min. Examination shows jugular venous distention to 8 cm. Bilateral basilar crackles are heard. Cardiac examination shows a diffuse, laterally displaced point of maximal impulse. There is a normal S1 and S2 and an S3. Examination shows 2+ pretibial edema bilaterally. An ECG shows no abnormalities. Echocardiography is most likely to show which of the following? A ) Asymmetric septal hypertrophy B ) Bicuspid aortic valve with stenosis C ) Diffuse hypokinesia and dilation of the ventricles D ) Dyskinesia of the left ventricular apex E ) Mitral valve prolapse 3. A 35-year-old man is brought to the emergency department because of intractable nausea and vomiting of nonbilious fluid over the past 48 hours. He has a history of duodenal ulcer disease treated with H2-receptor blocking agents. His temperature is 37 C (98.6 F), blood pressure is 90/60 mm Hg, pulse is 130/min, and respirations are 10/min. Examination shows mild epigastric tenderness. Which of the following are the most likely serum electrolyte findings? Na+ Cl K+ HCO3 (mEq/L) (mEq/L) (mEq/L) (mEq/L) A ) 115 80 4.0 25 B ) 140 80 2.5 40 C ) 145 100 5.0 15 D ) 150 105 2.5 25 E ) 160 135 5.0 25 4. A 3-year-old boy is brought for a follow-up examination. He just completed a 10-day course of amoxicillin that has not resolved his right ear pain. He appears irritable. His temperature is 38.9 C (102 F). Examination shows downward and lateral displacement of the right auricle with tenderness to palpation of the posterior auricular area; his neck is supple. Which of the following is the most appropriate next step in diagnosis?

A ) Bone scan B ) CT scan of the head C ) Tympanometry D ) Lumbar puncture E ) Tympanocentesis 5. A 67-year-old man has had shortness of breath on exertion for 3 months; he has had an 11.3-kg (25-lb) weight loss during this period. He has smoked two packs of cigarettes daily for 25 years. He appears chronically ill. Examination shows decreased breath sounds on the left; heart sounds are normal. An x-ray film of the chest shows a large left-sided pleural effusion. Which of the following is the most appropriate next step in diagnosis? A ) Bronchoscopy B ) Thoracoscopy C ) Closed pleural biopsy D ) Open pleural biopsy E ) Thoracentesis 6. A program for the primary prevention of coronary artery disease is implemented in a community in the USA. Assuming that diagnostic procedures and detection remain the same, which of the following measures involving the disease is most effective in monitoring the program? A ) Case fatality B ) Hospitalization C ) Incidence D ) Mortality E ) Prevalence 7. A 77-year-old woman comes to the physician because of low back pain for 3 months. She has hypertension controlled with a calcium-channel blocking agent and type 2 diabetes mellitus controlled with diet. Her vital signs are within normal limits. Examination shows no spinal or costovertebral angle tenderness; straight-leg raising produces pain in the low back at the L24 range. Knee jerk and ankle reflexes are 2+ bilaterally. Babinski's sign is absent bilaterally. Urinalysis shows 510 epithelial cells/hpf, 25 leukocytes/hpf, and few bacteria. Which of the following is the

most appropriate pharmacotherapy? A ) Acetaminophen B ) Gold C ) Methotrexate D ) Prednisone E ) Probenecid 8. A 32-year-old woman who is HIV positive has a CD4+ lymphocyte count of 800/mm3 (Normal 500). Her health maintenance regimen should include immunization against which of the following pathogens? A ) Haemophilus influenzae type b B ) Hepatitis A C ) Influenza virus D ) Neisseria meningitidis E ) Streptococcus pyogenes (group A) 9. A 32-year-old woman comes to the physician because of bright red rectal bleeding and severe stabbing pain with each bowel movement over the past 2 weeks. She has blood-streaked stools, and there is blood on the toilet paper. Over the past 2 months, she has had mild constipation with no change in the caliber of the stool. Examination shows a small anal fissure at the posterior midline. Rectal examination is painful, but no abnormalities are detected except for a small amount of bright red blood from the fissure. Which of the following is the most appropriate next step in management? A ) Anesthetic ointment and stool softeners B ) Anal dilatation under anesthesia C ) Debridement and closure of the fissure under anesthesia D ) Surgical flaps E ) Lateral internal sphincterotomy 10. A 28-year-old nulligravid woman comes for a routine health maintenance examination. She has had progressively severe dysmenorrhea over the past 6 months adequately controlled by nonsteroidal anti-inflammatory agents. Pelvic examination shows a normal vagina and cervix. The

uterus is retroverted and fixed, and there is nodularity of the cul-de-sac. A 6-cm left adnexal mass is palpated. Transvaginal ultrasonography shows a 7-cm septated adnexal mass. Four weeks later, there is no change in the size of the adnexal mass. Which of the following is the most appropriate diagnostic test? A ) Measurement of serum CA 125 level B ) Barium enema C ) CT scan of the pelvis D ) MRI of the pelvis E ) Laparoscopy 11. A 67-year-old woman is hospitalized because of abdominal pain and persistent copious vomiting for 24 hours. Two weeks ago, she was hospitalized for treatment of atrial fibrillation; after cardioversion to a normal sinus rhythm, she began treatment with warfarin. Yesterday at a followup visit, her INR was 6, and her medication was discontinued. She takes no other medications. Her temperature is 37 C (98.6 F), blood pressure is 100/78 mm Hg, pulse is 120/min and regular, and respirations are 20/min. The abdomen is distended and moderately tender; there is voluntary guarding in the epigastrium. There are no masses, organomegaly, or obvious hernias. Rectal examination shows no abnormalities. Test of the stool for occult blood is negative. Her hemoglobin level has decreased from 13 g/dL yesterday to 7.8 g/dL today. An ECG shows a normal sinus rhythm. Which of the following is the most likely explanation for this patient's abdominal symptoms? A ) Internal small-bowel herniation B ) Intestinal ischemia from a cardiac embolus C ) Intramural hematoma of the proximal small bowel D ) Intussusception of the small bowel E ) Malrotation of the small bowel 12. A 32-year-old man with alcoholism is brought to the emergency department by friends because he has been unable to stand without support and has had "funny eye movements"; they report that he has been drinking approximately 18 beers daily over the past month and has been increasingly confused over the past 5 days. He is awake and confused but is noncombative. His speech is slurred, and his breath smells of alcohol. His temperature is 37.2 C (99 F), blood pressure is 180/60 mm Hg, pulse is 110/min, and respirations are 18/min. Physical examination shows sixth cranial nerve palsy, horizontal diplopia, strabismus, and an asymmetric horizontal-gaze evoked nystagmus. Neurologic examination shows no focal weakness or numbness. When helped up and told to walk, he has a broad-based, uncertain gait. When asked how he arrived at the emergency department, he relates that "I drove to this place to visit some friends." The most likely cause of these findings is a deficiency of which of the following? A ) Folic acid B

) Magnesium C ) Vitamin B1 (thiamine) D ) Vitamin B12 (cyanocobalamin) E ) Zinc 13. A 32-year-old woman, gravida 2, para 2, comes to the physician because she has been amenorrheic for 4 months. Examination shows a well-estrogenized vagina and no evidence of virilization or other abnormalities. A serum pregnancy test is negative. She is given medroxyprogesterone and has the onset of bleeding 3 days later. Which of the following is the most likely cause of her condition? A ) Anovulation B ) Asherman's syndrome C ) Hypopituitarism D ) Menopause E ) Premature ovarian failure 14. A previously healthy 47-year-old woman comes to the emergency department because of a 36-hour history of nausea, vomiting, and abdominal pain that radiates to her back. Over the past 3 years, she has had intermittent episodes of cramping abdominal pain 1 to 2 hours after meals; the pain lasts for several hours and resolves spontaneously. She does not smoke and drinks one to two glasses of wine each evening. There is a family history of coronary artery disease and hypertension. Her temperature is 37 C (98.6 F), blood pressure is 100/60 mm Hg, pulse is 120/min, and respirations are 20/min. Abdominal examination shows moderate epigastric and right upper quadrant tenderness with no guarding or rebound; bowel sounds are decreased. Laboratory studies show: Hematocrit 45% Leukocyte count 9000/mm3 with a normal differential Serum Total bilirubin 1.5 mg/dL Alkaline phosphatase 120 U/L Aspartate aminotransferase (AST, GOT) 78 U/L Amylase 365 U/L Lipase 1223 U/L (N=1160) Triglycerides 300 mg/dL Which of the following is the most likely diagnosis? A ) Acute cholecystitis B ) Alcoholic hepatitis

C ) Alcoholic pancreatitis D ) Ascending cholangitis E ) Gallstone pancreatitis F ) Hepatitis A G ) Pancreatic cancer H ) Peptic ulcer disease I ) Triglyceride-induced pancreatitis 15. A healthy 37-year-old primigravid woman at 12 weeks' gestation comes for a routine prenatal visit. The pregnancy was achieved by in vitro fertilization. She does not use tobacco, alcohol, or drugs. She is a hematologist and works 10 to 12 hours daily. Two previous ultrasonographies have shown a triplet gestation. She weighs 66 kg (145 lb) and is 178 cm (70 in) tall. Her blood pressure is 116/70 mm Hg, and pulse is 72/min. Examination shows a uterus consistent in size with a 16week gestation. Her pelvis is normal-sized. This patient is at greatest risk for which of the following? A ) Abruptio placentae B ) Hepatitis B C ) HIV infection D ) Preterm labor E ) Uterine rupture 16. A 67-year-old man is brought to the emergency department because of a 3-day history of fever and headache. Five years ago, he underwent placement of a mechanical aortic valve for treatment of sequelae of rheumatic fever. He appears ill. His temperature is 40 C (104 F), blood pressure is 110/65 mm Hg, pulse is 110/min, and respirations are 22/min. A grade 3/6, systolic ejection murmur is heard. Neurologic examination shows mild left hemiparesis. Babinski's sign is present on the left. There is no nuchal rigidity. This patient is at greatest risk for which of the following complications? A ) Brain abscess B ) Carotid artery occlusion C ) Encephalitis

D ) Hydrocephalus E ) Venous sinus thrombosis 17. A 67-year-old woman comes for a routine health maintenance examination. She drinks 1 ounce of alcohol daily. Her blood pressure is 138/62 mm Hg, pulse is 76/min and regular, and respirations are 14/min. The lungs are clear to auscultation. The remainder of the examination shows no abnormalities. Laboratory studies show: Hemoglobin 12.8 g/dL Serum Ca2+ 11.9 mg/dL Creatinine 0.8 mg/dL Phosphorus 2.8 mg/dL Total protein 6.5 g/dL Albumin 4.2 g/dL Ionized calcium 5.8 mg/dL (N=4.55.1) Which of the following is the most appropriate next step in management? A ) Measurement of serum alkaline phosphatase activity B ) Measurement of serum parathyroid hormone level C ) Measurement of serum vitamin D level D ) Serum and urine protein electrophoresis E ) X-ray film of the chest F ) Skeletal survey G ) Bone scan The response options for the next two items are the same. You will be required to select one answer for each item in the set. For each patient with papilledema, select the most likely diagnosis. A ) Bacterial meningitis B ) Cerebral infarction

C ) Cryptococcal meningitis D ) Glioblastoma multiforme E ) Herpes simplex encephalitis F ) Hypertensive encephalopathy G ) Idiopathic intracranial hypertension H ) Intracerebral hemorrhage I ) St. Louis encephalitis 18. A 25-year-old woman comes to the emergency department because of increasingly severe bifrontal headaches over the past 6 months. During this period, she has had transient episodes of blindness lasting 1 to 2 seconds. She has not had nausea or vomiting. She has a long-standing history of difficulty losing weight. She currently weighs 113 kg (250 lb) and is 152 cm (60 in) tall. Her blood pressure is 120/80 mm Hg. Visual field testing shows enlarged blind spots. The remainder of the neurologic examination shows normal findings. A CT scan of the head with and without contrast shows no abnormalities. Examination of the cerebrospinal fluid shows: Opening pressure 300 mm H2O Glucose 70 mg/dL Protein 25 mg/dL WBC 1/mm3 RBC 0/mm3 Gram's stain and cultures are negative. For each patient with papilledema, select the most likely diagnosis. A ) Bacterial meningitis B ) Cerebral infarction C ) Cryptococcal meningitis D ) Glioblastoma multiforme E ) Herpes simplex encephalitis F ) Hypertensive encephalopathy G ) Idiopathic intracranial hypertension

H ) Intracerebral hemorrhage I ) St. Louis encephalitis 19. A 25-year-old man with a history of intravenous drug use comes to the emergency department because of a progressive diffuse headache, generalized malaise, and low-grade fever for 2 months. During this period, he has had a poor appetite resulting in a 6.8-kg (15-lb) weight loss. His temperature is 38 C (100.4 F). Examination shows neck stiffness. Mental status examination shows no abnormalities. Cranial nerve examination shows weakness of the lateral rectus muscle on the right and bilateral papilledema. A CT scan of the head with and without contrast shows moderate ventricular enlargement. Examination of cerebrospinal fluid shows: Opening pressure 220 mm H2O Glucose 35 mg/dL Protein 150 mg/dL WBC 100/mm3 Lymphocytes 100% RBC 1/mm 20. An asymptomatic 32-year-old woman comes for a routine health maintenance examination. Her mother and sister have a history of low back pain and disc herniation. The patient is a postal worker. She weighs 86 kg (190 lb) and is 165 cm (65 in) tall. Her blood pressure is 130/78 mm Hg, pulse is 74/min, and respirations are 16/min. Range of motion of the spine is normal and without pain. There is no scoliosis or excessive kyphosis of the back. Which of the following is the most effective strategy to decrease this patient's risk for developing low back pain? A ) Change in job B ) Limit physical activity C ) Stretching exercises D ) Weight-loss program E ) Nonsteroidal anti-inflammatory drug therapy F ) Prophylactic brace 21. Three days after being hospitalized for treatment of a hip fracture sustained in a fall, a 62-yearold woman becomes acutely short of breath and coughs up a small amount of blood-tinged sputum. She appears anxious. Her blood pressure is 110/70 mm Hg, pulse is 110/min, and respirations are 24/min. Examination shows no other abnormalities. Arterial blood gas analysis on 40% oxygen by face mask shows: pH 7.40 PCO2 38 mm Hg PO2 70 mm Hg Ventilation-perfusion lung scans show multiple segmental areas of mismatch on the right. Which of the following is the most appropriate next step in management? A ) Pulmonary angiography

B ) Dopamine therapy C ) Heparin therapy D ) Urokinase therapy E ) Intubation 22. A 24-year-old man is brought to the emergency department by police 1 hour after his ex-wife found him stumbling around in the yard. His blood pressure is 100/70 mm Hg, pulse is 90/min, and respirations are 16/min. The pupils are equal and reactive to light; the sclerae are injected. During the examination, he laughs without obvious reason, makes religious statements, and asks if there is anything to eat or drink. Mental status examination shows a broad range of affect; there is no evidence of thought disorder except for mild paranoia. Which of the following is the most appropriate next step in management? A ) Observation in the emergency department B ) Intramuscular administration of naloxone C ) Intravenous administration of 50% dextrose D ) Intravenous administration of lorazepam E ) Oral administration of chlorpromazine 23. A 17-year-old boy sustains a head injury and loses consciousness after diving into a freshwater lake from a platform. After being submerged for 3 minutes, he is rescued. He is initially cyanotic but then begins to cough and breathe spontaneously after a 5-minute resuscitation. Over the next 36 hours, which of the following complications is most likely? A ) Acute respiratory distress syndrome B ) Bacteremia C ) Bacterial pneumonia D ) Hypernatremia E ) Metabolic alkalosis 24. A 15-year-old boy is brought to the emergency department 30 minutes after being involved in a motor vehicle collision. He was the unrestrained passenger. On arrival, he is disoriented. His blood pressure is 80/40 mm Hg, pulse is 112/min, and respirations are 24/min. There is no jugular venous distention. Examination of the chest shows dullness to percussion and decreased breath sounds over the right hemithorax. Which of the following is the most likely diagnosis?

A ) Diaphragmatic rupture B ) Flail chest C ) Hemothorax D ) Massive aspiration E ) Pneumothorax 25. An 18-year-old man comes to the physician for an initial examination because of a 3-year history of fatigue and migrating joint pain. He has brought a large folder containing information about previous medical consultations, laboratory tests, and x-ray films. He takes no medications. He weighs 50 kg (110 lb) and is 173 cm (68 in) tall. Physical examination shows no other abnormalities. On mental status examination, he is preoccupied with his symptoms. When asked about his mood, he states that the future appears bleak, and that he is too tired to think about it. Which of the following is the most appropriate next step in management? A ) Ask about further symptoms of obsessions and compulsions B ) Ask about sexual history including sexual orientation and practices C ) Ask about suicidal feelings D ) Ask about travel history over the past 6 months E ) Obtain a detailed exercise history F ) Measurement of serum Lyme (Borrelia burgdorferi) antibody level G ) Urine toxicology screening 26. An 80-year-old woman has had bleeding gums for 3 weeks. Her diet has consisted of only tea and toast for 2 years. She appears thin and weak. Vital signs are normal. Her gums are hypertrophied and exude blood with pressure. There are ecchymoses of the inner thighs and small hemorrhages around the hair follicles. The remainder of the examination shows normal findings. Supplementation with which of the following vitamins is most likely to have prevented this condition? A )A B ) B12 (cyanocobalamin) C )C D )D

E )E 27. A 30-year-old woman, gravida 2, para 1, is brought to the emergency department in labor. An episiotomy is performed. Following delivery of the head, the shoulders do not follow with the usual traction and maternal pushing. Which of the following is the most appropriate next step in management? A ) Flexing the woman's knees toward her shoulders B ) More forceful traction and fundal pressure C ) Delivering the posterior arm D ) Rotating the head 180 degrees E ) Symphysiotomy 28. A 32-year-old man comes to the physician because of a 3-day history of low back pain, urinary hesitancy, and pain with urination. He has had several similar episodes over the past 3 years. He has not had any urethral discharge or recent sexual contacts. His temperature is 37.2 C (98.9 F), and blood pressure is 126/76 mm Hg. The lungs are clear to auscultation; there is no costovertebral angle tenderness. Abdominal examination shows no tenderness or masses. There is no tenderness to palpation of the lower back. Straight-leg raising to 90 degrees is negative. There are no motor or sensory deficits in the lower extremities. Rectal examination shows a diffuse, minimally enlarged, tender prostate with no masses. Laboratory studies show: Serum prostate-specific antigen 6 ng/mL (N<4) Urine Specific gravity 1.020 Blood negative Glucose negative Ketones negative Leukocyte esterase negative Nitrites negative Which of the following is the most likely diagnosis? A ) Benign prostatic hypertrophy B ) Cystitis C ) Epididymitis D ) Prostate cancer E ) Prostatitis

F ) Pyelonephritis G ) Urolithiasis 29. A 16-year-old boy with neurofibromatosis is brought for a follow-up examination. His uncle also has neurofibromatosis. He has a 1-year history of headaches during which his parents say he appears pale. Six months ago, he underwent operative treatment for an optic nerve glioma. His blood pressure is 164/105 mm Hg, pulse is 102/min, and respirations are 14/min. The thyroid glands are not enlarged. No murmurs are heard, and radial pulses are equal. Abdominal examination shows no abnormalities. Which of the following is the most likely cause of this patient's high blood pressure? A ) Catecholamine-producing tumor B ) Carcinoma of the thyroid gland C ) Essential hypertension D ) Overproduction of aldosterone from an adrenal adenoma E ) Postsubclavian coarctation of the aorta 30. A 14-month-old girl is brought to the physician because of a 14-hour history of irritability and episodes of drawing her knees toward her chest. During this period, she has vomited nonbilious fluid twice and had a bowel movement containing a small amount of blood. She had an upper respiratory tract infection 2 weeks ago. She is listless except for intermittent episodes of discomfort. Her temperature is 38 C (100.4 F). Abdominal examination shows right-sided tenderness without guarding or rebound; bowel sounds are present. Rectal examination shows bright red blood and mucus. An x-ray film of the abdomen shows no abnormalities. Which of the following is the most appropriate next step in management? A ) X-ray film of the upper gastrointestinal tract with contrast B ) Water-soluble contrast enema C ) Corticosteroid enemas D ) Admission to the hospital for total parenteral nutrition E ) Immediate laparotomy 31. An asymptomatic 47-year-old man comes for a preemployment examination. He has never been hospitalized. He is a computer programmer, and he plays handball once weekly. His maternal grandmother had type 2 diabetes mellitus, and a paternal uncle had heart disease. The patient's blood pressure is 126/80 mm Hg. Examination shows no abnormalities. His total serum cholesterol level is 225 mg/dL. Which of the following is the most appropriate next step in management? A ) Step 2 American Heart Association cardiac diet

B ) Serum lipid studies while fasting C ) Exercise stress test D ) Oral cholestyramine and niacin therapy E ) Oral pravastatin therapy at bedtime 32. A 25-year-old woman comes to the physician because of a 3-month history of the unexplained urge to eat a few tablespoons of cornstarch daily. The amount of cornstarch ingested has increased gradually during this time. She has leiomyomata uteri; she is otherwise healthy. Her weight is unchanged from her last visit 1 year ago; she weighs 61 kg (135 lb) and is 168 cm (66 in) tall. Her blood pressure is 120/80 mm Hg, and pulse is 100/min. Physical examination shows mild pallor. Neurologic examination shows no abnormalities. She is embarrassed about her problem. She has no compulsive behavior or obsessive thoughts. An ECG shows sinus tachycardia. Which of the following is the most appropriate next step in management? A ) Psychiatric assessment B ) Complete blood count C ) Thyroid function studies D ) Urine toxicology screening E ) Admission to an eating disorders clinic F ) Antipsychotic therapy G ) Folic acid supplementation H ) Selective serotonin reuptake inhibitor therapy 33. A 30-year-old woman comes to the physician for a follow-up examination 4 months after starting treatment with lithium carbonate; during this period she has had a 3.2-kg (7-lb) weight gain. Her medication was prescribed soon after the birth of her son to treat racing thoughts, increased spending, and overtalkativeness; she has had difficulty sleeping for more than 2 hours nightly. Her symptoms decreased after 2 weeks of lithium carbonate therapy. She continues to take lithium carbonate (300 mg three times daily). Her temperature is 37 C (98.6 F), blood pressure is 120/80 mm Hg, and pulse is 70/min. Examination shows normal findings. The most appropriate next step is measurement of which of the following? A ) Fasting serum glucose level B ) Leukocyte count C ) Serum alkaline phosphatase activity

D ) Serum creatinine level E ) Serum thyroid-stimulating hormone level 34. On a routine examination, a 2-year-old boy has a hemoglobin level of 10.5 g/dL, hematocrit of 30%, and mean corpuscular volume of 72 m3. He drinks four to five 8-oz bottles of milk daily. He maintains a regular diet but does not eat vegetables. Which of the following is the most likely diagnosis? A ) Folic acid deficiency B ) Iron deficiency C ) Sickle cell disease D ) Thalassemia E ) Vitamin B6 deficiency 35. A 57-year-old man is brought to the physician by his wife because his skin has appeared yellow for 3 weeks. Examination shows jaundice and scleral icterus. His total serum bilirubin level is 8 mg/dL with a direct component of 6.2 mg/dL. A CT scan of the abdomen shows a large lesion in the head of the pancreas. When the results are initially discussed, the patient says that he does not want to hear the report, and his wife agrees to abide by his wishes. Which of the following is the most appropriate course of action? A ) Withhold the results as the patient wishes B ) Contact the patient's children to discuss the results C ) Consult with the hospital ethics committee D ) Insist on telling the patient the results E ) Refer him to another physician 36. A previously healthy 52-year-old man comes to the physician because of a 3-month history of increased urinary volume and increased urinary frequency at night. He has had a 6.8-kg (15-lb) weight loss during this period despite no change in appetite. His father has hypertension, and his mother has hypertension and type 2 diabetes mellitus. He currently weighs 95 kg (210 lb) and is 178 cm (70 in) tall. His blood pressure is 160/85 mm Hg in both arms. Examination shows no other abnormalities. His nonfasting serum glucose level is 280 mg/dL. Which of the following serum levels is most likely to be increased in this patient? A ) Bicarbonate B ) Glucagon

C ) HDL-cholesterol D ) Insulin E ) Ketones 37. Three days after hospitalization for heparin treatment of deep venous thrombosis of a left superficial femoral vein, a 52-year-old woman has prolonged bleeding from a venipuncture site. She had a pulmonary embolus 2 years ago. Her temperature is 37.5 C (99.5 F). Examination shows multiple ecchymoses at the venipuncture sites with oozing of fresh blood. Laboratory studies show: Hemoglobin 10.5 g/dL Platelet count 25,000/mm3 Prothrombin time 14 sec (INR=1.3) Partial thromboplastin time 65 sec Plasma fibrinogen 300 mg/dL (N=200400) Serum creatinine 1.1 mg/dL Which of the following is the most likely diagnosis? A ) Adverse drug reaction B ) Disseminated intravascular coagulation C ) Excessive blood loss from anticoagulation D ) Factor VIII deficiency E ) Factor IX deficiency F ) Immune thrombocytopenic purpura G ) Thrombotic thrombocytopenic purpura 38. A 37-year-old woman comes to the physician because of an itchy rash over her trunk for 2 weeks. She has not had fever, chills, shortness of breath, chest pain, or gastrointestinal symptoms. She has a history of recurrent urinary tract infections and has been taking trimethoprimsulfamethoxazole prophylaxis for the past year. She is in mild distress. Her temperature is 37.5 C (99.5 F), blood pressure is 96/62 mm Hg, pulse is 78/min, and respirations are 14/min. Examination shows a maculopapular erythematous rash over the trunk. Laboratory studies show: Leukocyte count 10,500/mm3 Segmented neutrophils 72% Bands 1% Eosinophils 15%

Lymphocytes 4% Monocytes 8% Serum Urea nitrogen (BUN) 12 mg/dL Creatinine 0.9 mg/dL Urine WBC 2/hpf RBC 2/hpf Which of the following is the most likely cause of these findings? A ) Eczema B ) Medication adverse effect C ) Staphylococcal skin infection D ) Streptococcal skin infection E ) Urinary tract infection 39. A 19-year-old primigravid woman at 34 weeks' gestation comes to the physician for a routine prenatal visit. Her pregnancy has been uncomplicated. She has no history of serious illness. She takes no medications and has no known allergies. Examination shows a uterus consistent in size with a 34-week gestation. A routine clean-catch urine culture grows greater than 100,000 colonies/mL of Escherichia coli. Which of the following is the most appropriate pharmacotherapy? A ) Ampicillin B ) Ciprofloxacin C ) Clindamycin D ) Doxycycline E ) Trimethoprim-sulfamethoxazole 40. A 13-year-old girl is brought for a well-child examination. Menses have occurred every other month since menarche 10 months ago. Her last menstrual period was 1 week ago. She is not sexually active. Sexual development is Tanner stage 3. Examination shows no abnormalities. Which of the following is the most appropriate next step in management? A ) Discussion of pregnancy prevention B ) Measurement of serum luteinizing and follicle-stimulating hormone levels C ) Urine -hCG test D

) Pelvic examination E ) Estrogen therapy F ) Diagnostic laparoscopy 41. A 72-year-old man comes to the emergency department after a 5-minute episode of blindness in the right eye. Over the past month, he has had headache and pain in the jaw with chewing. His temperature is 38.1 C (100.6 F). Examination shows tender, nodular temporal arteries with decreased pulses. His erythrocyte sedimentation rate is 92 mm/h. Which of the following is the most appropriate immediate step in management? A ) Carotid ultrasonography B ) Aspirin therapy C ) Cefotaxime therapy D ) Corticosteroid therapy E ) Temporal artery biopsy 42. A 42-year-old man comes for a routine health maintenance examination. There is no family history of coronary artery disease, and he does not smoke. His weight is appropriate for his height. His blood pressure is 120/80 mm Hg. Serum lipid studies show a total cholesterol level of 190 mg/dL, HDL-cholesterol level of 40 mg/dL, and triglyceride level of 150 mg/dL. Which of the following is the most appropriate next step in management? A ) Recommend the Step 2 National Cholesterol Education Program diet B ) Measure serum LDL-cholesterol level now C ) Measure total serum cholesterol level in 5 years D ) Prescribe prophylactic aspirin E ) Begin treatment with lovastatin 43. A 28-month-old boy has a history of cyanosis since birth with episodes of syncope. Examination shows cyanosis and clubbing. The lungs are clear to auscultation. There is a right ventricular heave, a systolic click, a single S2, and a grade 3/6 systolic murmur. Which of the following is the most likely diagnosis? A ) Bicuspid aortic valve B ) Coarctation of the aorta C

) Mitral stenosis D ) Patent ductus arteriosus E ) Tetralogy of Fallot 44. A healthy 18-year-old woman comes for a routine health maintenance examination. Menses occur at regular 28-day intervals. Her last menstrual period was 2 weeks ago. She is sexually active with one partner, and she and her partner use condoms inconsistently for contraception. Her maternal grandmother was diagnosed with breast cancer at the age of 65 years, her paternal grandfather was diagnosed with colon cancer at the age of 72 years, and her maternal grandfather died of a myocardial infarction at the age of 66 years. Examination shows no abnormalities. Which of the following is the most appropriate screening test for this patient? A ) Complete blood count B ) Fasting serum lipid studies C ) Test of the stool for occult blood D ) Testing for Neisseria gonorrhoeae and Chlamydia trachomatis E ) Urinalysis The response options for the next two items are the same. You will be required to select one answer for each item in the set. For each child with fever and cough, select the most likely diagnosis. A ) 1-Antitrypsin deficiency B ) Cystic fibrosis C ) Pneumothorax D ) Pulmonary alveolar proteinosis E ) Pulmonary aspergillosis F ) Pulmonary hemorrhage G ) Pulmonary tuberculosis 45. A 20-month-old girl is brought to the physician because of fever and cough for 2 days. She has had several similar episodes since the age of 4 months. Three months ago, she and her family visited her grandmother in Finland for 2 weeks. She is at the 25th percentile for length and 5th percentile for weight. She appears thin and pale. Her temperature is 38 C (100.4 F), pulse is

150/min, and respirations are 40/min. Examination shows mild clubbing. Wheezing and bilateral crackles are heard at the lung bases. An x-ray film of the chest shows streaky densities bilaterally with mild hyperinflation. For each child with fever and cough, select the most likely diagnosis. A ) 1-Antitrypsin deficiency B ) Cystic fibrosis C ) Pneumothorax D ) Pulmonary alveolar proteinosis E ) Pulmonary aspergillosis F ) Pulmonary hemorrhage G ) Pulmonary tuberculosis 46. A previously healthy 16-year-old boy is brought to the physician because of fever and cough with right-sided chest pain for 2 weeks. Six months ago, he visited his grandparents in Albania for 2 weeks. He weighs 54 kg (120 lb) and is 173 cm (68 in) tall. He appears thin and pale. His temperature is 38.2 C (100.8 F), pulse is 76/min, and respirations are 36/min. Examination shows shallow respirations with decreased breath sounds at the right lung base. An x-ray film of the chest shows a right pleural effusion and hilar adenopathy. _______________________________________________________________________________ ___________________________________________________ Section 4:-1. A previously healthy 52-year-old man comes to the emergency department because of hiccups for 1 week. He has smoked two packs of cigarettes daily for 30 years. He does not drink alcohol. He is alert and oriented. His temperature is 37 C (98.6 F), blood pressure is 150/95 mm Hg, pulse is 70/min, and respirations are 12/min. Physical and neurologic examinations show no abnormalities. His serum sodium level is 120 mEq/L. An x-ray film of the chest shows a right hilar mass. Which of the following is the most appropriate next step in treatment? A ) Bisphosphonate therapy B ) Calcitonin therapy C ) Calcium therapy D ) Dexamethasone therapy E ) 5% Dextrose in 0.225% saline therapy F

) 5% Dextrose in 0.45% saline therapy G ) 5% Dextrose in water therapy H ) Fluid restriction I ) Hydrocortisone therapy J ) Lactated Ringer's solution K ) Mannitol therapy L ) Potassium therapy M ) 0.9% Saline therapy N ) 3% Saline therapy O ) Sodium bicarbonate therapy 2. A 67-year-old man comes to the physician because of a 6-month history of double vision, slurred speech, and difficulty swallowing liquids. Initially, his symptoms were intermittent but now occur daily and are worse at the end of the day. Over the past week, he has had shortness of breath with exertion. He has pernicious anemia treated with monthly vitamin B12 (cyanocobalamin) injections and autoimmune thyroid disease currently treated with thyroid replacement therapy. Examination shows bilateral ptosis and disconjugate gaze. There is bilateral facial weakness and hypernasal speech. The tongue is weak, and the gag reflex is reduced. Muscle strength is 4/5 in the proximal muscles of the upper and lower extremities. Deep tendon reflexes are 2+ diffusely. Babinski's sign is absent. Sensory examination shows no abnormalities. A CT scan of the chest is shown. Which of the following is the most likely diagnosis of this patient's intrathoracic lesion? A ) Aspiration pneumonia B ) Bronchogenic carcinoma C ) Lung metastasis D ) Sarcoidosis E ) Thymoma F ) Thyroid carcinoma 3. A 32-year-old man receiving intensive chemotherapy for Hodgkin's disease has a temperature of 39 C (102.2 F). His respirations are 40/min. Widespread crackles are heard in all lung fields. An xray film of the chest shows a diffuse alveolar and interstitial pattern. Which of the following is the most likely causal organism?

A ) Aspergillus species B ) Candida albicans C ) Coccidioides immitis D ) Pneumocystis carinii E ) Streptococcus pneumoniae 4. A 20-year-old African American man with sickle cell disease comes to the physician because of a 1-week history of shortness of breath on exertion, fatigue, and generalized weakness. He has had no fever, chills, night sweats, or cough productive of sputum. His only medication is oxycodone for joint pain. He weighs 68 kg (150 lb) and is 168 cm (66 in) tall. His temperature is 36.7 C (98 F), blood pressure is 120/70 mm Hg, pulse is 76/min, and respirations are 18/min. Examination shows no abnormalities. Laboratory studies show: Hematocrit 20% Mean corpuscular volume 110 m3 Leukocyte count 2300/mm3 Reticulocyte count 1.8% Which of the following is the most likely mechanism for these findings? A ) Adverse drug reaction B ) Atrophy of gastric mucosa C ) Bacterial overgrowth in the small intestine D ) Increased demand for folic acid E ) Increased demand for vitamin B12 (cyanocobalamin) F ) Malabsorption G ) Vitamin B1 (thiamine) deficiency 5. A 44-year-old woman comes to the emergency department 2 days after being discharged from the hospital for abdominal pain; an exploratory laparoscopy showed no abnormalities. Over the past 5 years, she had been admitted to the hospital numerous times for the evaluation of a variety of symptoms; all work-ups had been negative. Her temperature is 39.2 C (102.6 F). Examination shows a reddened, indurated laparoscopic wound. She is admitted to the hospital and given intravenous antibiotics. On the second hospital day, a nurse witnesses the patient rubbing saliva into her laparoscopy site. Which of the following is the most likely diagnosis? A ) Adjustment disorder with disturbance of conduct

B ) Conduct disorder C ) Conversion disorder D ) Factitious disorder E ) Hypochondriasis F ) Major depressive disorder G ) Oppositional defiant disorder H ) Somatization disorder 6. A 37-year-old woman is brought to the emergency department because she has been unable to see out of her right eye since awakening 2 hours ago. She states that any movement of the eye is painful. Examination shows visual acuity of 20/200 in the right eye and 20/20 in the left eye. The optic fundi are normal. The left pupil reacts normally to light. The right pupil is poorly reactive to direct light. The remainder of the eye examination shows no abnormalities. Which of the following is the most likely site of the lesion? A ) Left optic nerve B ) Left optic radiation C ) Left optic tract D ) Left visual cortex E ) Optic chiasm F ) Retina G ) Right optic nerve H ) Right optic radiation I ) Right optic tract J ) Right visual cortex The response options for the next two items are the same. You will be required to select one answer for each item in the set. For each patient with peripheral nerve dysfunction, select the most likely site of nerve injury.

A ) Axillary nerve B ) Cervical nerve root at the cervical foramen C ) Long thoracic nerve D ) Median nerve above the elbow E ) Median nerve at the elbow F ) Median nerve at the wrist G ) Musculocutaneous nerve above the elbow H ) Musculocutaneous nerve at the elbow I ) Musculocutaneous nerve at the wrist J ) Radial nerve above the elbow K ) Radial nerve at the elbow L ) Radial nerve at the wrist M ) Suprascapular nerve N ) Thoracodorsal nerve O ) Ulnar nerve above the elbow P ) Ulnar nerve at the elbow Q ) Ulnar nerve at the wrist 7. A previously healthy 42-year-old carpenter comes to the physician because of a 6-month history of pain and numbness in his dominant hand that awakens him at night. He describes numbness in his long and index fingers after driving for extended periods of time. Examination shows minimal atrophy of the thenar muscles. There is normal sensation to light touch on the little finger and the palm of the right hand. Sensation to light touch is decreased at the tip of the thumb, index finger, and long finger. X-ray films of the right elbow and wrist show no abnormalities.

For each patient with peripheral nerve dysfunction, select the most likely site of nerve injury. A ) Axillary nerve B ) Cervical nerve root at the cervical foramen C ) Long thoracic nerve D ) Median nerve above the elbow E ) Median nerve at the elbow F ) Median nerve at the wrist G ) Musculocutaneous nerve above the elbow H ) Musculocutaneous nerve at the elbow I ) Musculocutaneous nerve at the wrist J ) Radial nerve above the elbow K ) Radial nerve at the elbow L ) Radial nerve at the wrist M ) Suprascapular nerve N ) Thoracodorsal nerve O ) Ulnar nerve above the elbow P ) Ulnar nerve at the elbow Q ) Ulnar nerve at the wrist 8. A previously healthy 37-year-old man comes to the physician because of a 2-month history of pain in the forearm and little finger of his dominant hand; he has been working as a receptionist for 6 months. He describes numbness in his little finger and weakness of his grip. There is decreased sensation to light touch at the tip of the little finger. Paresthesias are elicited with compression of the cubital tunnel.

9. A 3-month-old boy is brought for a well-child examination. He has poor head control. Examination shows generalized hypotonia. The point of maximal impulse is at the left anterior axillary line. The liver edge is palpated 4 cm below the right costal margin. The spleen is not palpable. Which of the following is the most likely diagnosis? A ) Congenital muscular dystrophy B ) Glycogen storage disease, type II (Pompe's disease) C ) GM1 gangliosidosis D ) Infant botulism E ) Ventricular septal defect 10. Over the past 3 months, a 30-year-old woman has had intermittent episodes of headache, palpitations, sweating, and irritability. Her blood pressure is 150/100 mm Hg while supine and 149/100 mm Hg while standing, and pulse is 90/min while supine and 110/min while standing. Examination shows no abnormalities except for pallor. Which of the following is the most likely location of this patient's lesion? A ) Adrenal gland B ) Brain C ) Heart D ) Kidney E ) Thyroid gland 11. A 19-year-old man is brought to the emergency department 45 minutes after sustaining a single, large stab wound to the right upper quadrant of the abdomen. He is obtunded. His blood pressure is 60/palpable mm Hg, and pulse is 148/min. Breath sounds are equal bilaterally. Examination shows a 4-cm laceration in the right upper quadrant in the midclavicular line. The abdomen is distended. Which of the following is the most appropriate next step in management? A ) X-ray films of the abdomen and pelvis B ) Angiography C ) Contrast study of the wound tract D ) CT scan of the abdomen E ) CT scan of the chest

F ) Laparoscopy G ) Peritoneal lavage H ) Laparotomy I ) Local wound exploration 12. A 52-year-old woman comes to the physician because of decreased libido; this symptom began 8 months ago, after she underwent a total abdominal hysterectomy and bilateral salpingooophorectomy for leiomyomata uteri and menorrhagia. She has been taking hormone replacement therapy with conjugated estrogen since the operation. Examination shows a moist, rugated vagina. Which of the following is the most likely cause of these findings? A ) Decreased androgens B ) Decreased estrogen C ) Decreased follicle-stimulating hormone (FSH) D ) Decreased luteinizing hormone (LH) E ) Decreased progesterone F ) Decreased prolactin G ) Increased androgens H ) Increased estrogen I ) Increased FSH J ) Increased LH K ) Increased progesterone L ) Increased prolactin 13. A 7-day-old newborn is brought for a well-child examination. He was born at home. His mother has taken acetaminophen for perineal discomfort. The newborn is breast-feeding well. He is at the 75th percentile for length and weight. Examination shows no abnormalities. After vaccination with hepatitis B, he has prolonged bleeding at the injection site. There is no family history of excessive bleeding. Laboratory studies show:

Hemoglobin 16 g/dL Leukocyte count 6800/mm3 Segmented neutrophils 48% Bands 2% Lymphocytes 50% Platelet count 280,000/mm3 Prothrombin time 20 sec (INR=1.6) Partial thromboplastin time 60 sec Which of the following is the most likely mechanism for these findings? A ) Autoimmunization B ) Bacterial toxic effect C ) Factor VIII deficiency D ) Factor IX deficiency E ) Immunoglobulin deficiency F ) Iron deficiency G ) Pharmacologic effect H ) Viral toxic effect I ) Vitamin deficiency J ) Zinc deficiency 14. A 67-year-old man with long-standing signs and symptoms of congestive heart failure is admitted to the hospital because of progressive shortness of breath. Examination shows no other abnormalities. An x-ray film of the chest shows cardiomegaly, cephalization of blood vessels, and a right-sided pleural effusion. Which of the following sets of pleural fluid findings is most likely in this patient? Leukocyte Segmented Protein Glucose count neutrophils Monocytes (g/dL) (mg/dL) (/mm3) (%) (%) A ) 2.5 10

10,000 50 50 B ) 2.5 90 2000 60 40 C ) 3.8 40 30,000 80 20 D ) 4.5 60 10,000 20 80 E )

4.5 90 2000 50 50 15. For 8 weeks, a 52-year-old man with a 5-year history of type 2 diabetes mellitus has had deep burning pain in the ball of his right foot and big toe when the foot is raised above chest level. He also has cramping in his right calf when he walks more than 50 feet. He has smoked two packs of cigarettes daily for 30 years. Femoral pulses are palpable; pedal pulses are absent. Which of the following is the most likely diagnosis? A ) Aortoiliac stenosis B ) Femoral popliteal stenosis C ) Mononeuropathy D ) Vasculitis E ) Venous stasis 16. A 1-year study of a new drug to treat hypertension is conducted. One hundred patients with hypertension are enrolled; 50 patients are given the new drug and another 50 patients are given hydrochlorothiazide. All patients completed the trial. One noted unexpected effect is increased growth of scalp hair which occurred in those taking the new drug, a nonstatistically significant difference (p>0.10). This effect has also been reported in studies of other similar drugs in the new therapeutic class. The investigators of the study concluded that the new drug did not cause hair growth. Which of the following features of this study is most likely to affect the validity of this conclusion? A ) Differential follow-up B ) Lead time bias C ) Length of the study D ) Sample size E ) Self-selection 17. A 38-year-old woman, gravida 2, para 1, at 38 weeks' gestation has had no fetal movement for 36 hours. Her prenatal course, prenatal tests, and fetal growth have been normal. Fetal heart tones

are heard by Doppler. Which of the following is the most appropriate next step in management? A ) Routine prenatal visit in 1 week B ) Maternal hydration C ) Nonstress test D ) Immediate induction of labor E ) Amniocentesis 18. A healthy 7-year-old boy is brought to the physician 1 week after he was exposed for several hours to a child with chickenpox. The patient and his healthy sister have not had chickenpox. They have not received varicella vaccine. Which of the following is the most appropriate management for the patient and his sister at this time? A ) Administer acyclovir as prophylaxis B ) Administer immune globulin, intravenously C ) Administer aspirin therapy if vesicles appear D ) Advise the parents to keep the siblings home from school to prevent exposing their classmates E ) Inform the parents that a vesicular rash may appear at any time over the next 2 weeks 19. A 6-year-old boy is brought to the physician by his mother because of progressive visual loss over the past year. Over the past 2 years, he has had deterioration of his hearing, speech, writing, and intellectual performance. His maternal uncle had similar symptoms. Visual acuity is 20/200 bilaterally. Funduscopic examination shows optic atrophy. His hearing is markedly impaired. There is weakness and spasticity of all extremities. Deep tendon reflexes are extremely hyperactive. Babinski's sign is present bilaterally. On mental status examination, he is not oriented to place, year, month, or the names of his siblings. An MRI of the brain shows marked symmetric white matter disease involving all lobes. Diagnostic studies are most likely to show which of the following? A ) Abnormally decreased serum cholesterol level B ) Acanthocytes on blood smear C ) An excess of very long chain fatty acids D ) Normal nerve conduction studies E ) Vitamin E deficiency 20. A 22-year-old primigravid woman at 16 weeks' gestation is brought to the emergency

department because of progressive shortness of breath over the past 48 hours. Her temperature is 37 C (98.6 F), blood pressure is 120/70 mm Hg, pulse is 100/min, and respirations are 24/min. Scattered wheezes are heard. Pelvic examination shows a uterus that extends to the umbilicus. Fetal heart tones are absent. Her hematocrit is 32%, leukocyte count is 11,000/mm3, and serum hCG level is 300,000 mIU/mL. Pulse oximetry on room air shows an oxygen saturation of 92%. An x-ray film of the chest shows multiple round densities throughout all lung fields. Which of the following is the most likely diagnosis? A ) Bacterial pneumonia B ) Choriocarcinoma C ) Pulmonary embolism D ) Tuberculosis E ) Viral pneumonia 21. A 42-year-old man comes to the physician because of progressive swelling of the legs over the past 2 months. He has a history of stage IIA Hodgkin's disease treated 1 year ago with radiation therapy to the neck and chest. His temperature is 37 C (98.6 F), blood pressure is 102/80 mm Hg, pulse is 110/min, and respirations are 22/min. Examination shows jugular venous distention that increases with inspiration. The lungs are clear to auscultation. Cardiac examination shows a nondisplaced point of maximal impulse; heart sounds are distant. An early diastolic sound is heard at the apex. Abdominal examination shows mild distention with shifting dullness. The liver is pulsatile, and its edge is palpated 4 cm below the right costal margin. There is 2+ peripheral edema extending up to the knees. Which of the following is the most likely mechanism of this patient's increased central venous pressure? A ) Constrictive pericarditis B ) Cor pulmonale C ) Left-sided congestive heart failure D ) Mitral stenosis E ) Tricuspid stenosis 22. A 5-year-old boy is brought to the emergency department 30 minutes after he fainted at home after standing up from a sitting position. His symptoms began 3 days ago with diarrhea and vomiting. He has had no urine output for 18 hours. He is alert but quiet. His temperature is 37.5 C (99.5 F), blood pressure is 75/45 mm Hg, pulse is 120/min, and respirations are 28/min. Examination shows dry lips and tenting of the skin. There is no abdominal tenderness. Bowel sounds are hyperactive. The remainder of the examination shows no abnormalities. His capillary refill time is 5 seconds. Intravenous bolus doses of 0.9% saline are administered. Bladder catheterization yields 5 mL of urine. Urinalysis is most likely to show which of the following? A ) Blood B

) Erythrocyte casts C ) Hyaline casts D ) Leukocyte casts E ) Oxalate crystals 23. A 2-year-old boy is brought to the physician because of fever and cough for 2 days. He had Streptococcus pneumoniae meningitis at the age of 1 year, S. pneumoniae bacteremia at the age of 18 months, and pneumonia at the age of 22 months. Two maternal uncles died before the age of 2 years from "infection." His temperature is 39.8 C (103.6 F), pulse is 150/min, and respirations are 60/min. Examination shows subcostal retractions on inspiration. Laboratory studies show: Hemoglobin 10 g/dL Leukocyte count 36,000/mm3 Segmented neutrophils 70% Bands 20% Lymphocytes 8% Monocytes 2% Platelet count 240,000/mm3 Serum IgA <5 mg/dL IgG 30 mg/dL IgM <5 mg/dL An x-ray film of the chest shows an infiltrate in the left upper lobe. Which of the following is the most appropriate next step in management? A ) Reexamination in 12 weeks B ) Prednisone therapy only C ) Prednisone, vincristine, doxorubicin, asparaginas e, and methotrexate therapy D ) Zidovudine (AZT), lamivudine (3TC), and ritonavir therapy E ) Intravenous immunoglobulin infusion F ) Bone marrow transplantation G ) Thymus transplantation 24. A previously healthy 13-year-old girl is brought to the physician because of a 2-month history of intermittent abdominal pain and loose stools. She has had a 2.3-kg (5-lb) weight loss during this period due to a decreased appetite, but she drinks up to 1 liter of fruit juice daily. At her last visit 9

months ago, she was at the 50th percentile for height and the 50th percentile for weight. She is now at the 50th percentile for height and the 10th percentile for weight. Her temperature is 37.8 C (100 F), pulse is 80/min, and respirations are 18/min. Examination shows no other abnormalities. Test of the stool for occult blood is positive. Laboratory studies show: Hemoglobin 8.5 g/dL Leukocyte count 9100/mm3 Segmented neutrophils 55% Lymphocytes 35% Monocytes 10% Platelet count 650,000/mm3 Erythrocyte sedimentation rate 75 mm/h Serum Na+ 139 mEq/L Cl 101 mEq/L K+ 3.2 mEq/L HCO3 23 mEq/L Urea nitrogen (BUN) 8 mg/dL Glucose 73 mg/dL Creatinine 0.2 mg/dL A barium enema shows patches of ulcerations along the proximal colon with reflux of dye into the terminal ileum. Which of the following is the most appropriate next step in management? A ) Eliminate fruit juice from the patient's diet B ) Switch to a gluten-free diet C ) H2- receptor blocking agent therapy D ) Prednisone and aminosalicylate therapy E ) Bowel resection 25. An asymptomatic 57-year-old man comes to the physician for a routine health maintenance examination. He has smoked one pack of cigarettes daily for 37 years. His blood pressure is 180/112 mm Hg, and pulse is 82/min. Abdominal examination shows a bruit in the right upper quadrant and no masses. His hematocrit is 42%, serum urea nitrogen (BUN) level is 23 mg/dL, and serum creatinine level is 1.4 mg/dL. Which of the following is the most likely cause of this patient's bruit? A ) Accumulation of lipids in the arterial wall B ) Hypertrophy of the arterial wall media C ) Infiltration of arterial wall by giant cells D ) Infiltration of round cells in the arterial wall E

) Reflex vasodilation 26. An otherwise healthy 4-month-old girl is brought to the physician because of a birthmark on her arm that has increased in size over the past several weeks. Examination shows a 3 x 2-cm, bright red, raised, soft, nontender, compressible patch over the left forearm. Which of the following is the most appropriate next step in management? A ) Topical corticosteroid therapy B ) Biopsy C ) Laser therapy D ) Excision E ) No intervention is necessary 27. A 7-year-old girl is brought to the physician in September because of fever and sore throat for 1 day. She is in the third week of second grade. Her temperature is 38.6 C (101.5 F). Examination shows an erythematous pharynx and slightly enlarged tonsils without exudate. There is no significant cervical lymphadenopathy. A rapid test for group A streptococcus is negative. Which of the following is the most appropriate next step in management? A ) Monospot test B ) Throat culture C ) Intramuscular penicillin therapy D ) Oral erythromycin therapy E ) Oral penicillin therapy The response options for the next two items are the same. You will be required to select one answer for each item in the set. For each patient with weakness, select the most appropriate test to establish the cause of the condition. A ) Brain stem auditory evoked potentials B ) Carotid ultrasonography C ) Chromosomal analysis for trinucleotide repeat D ) Electroencephalography

E ) Electromyography and nerve conduction studies F ) MRI of the spine G ) Muscle biopsy H ) Repetitive nerve stimulation I ) Somatosensory evoked potentials J ) SPECT scan K ) Temporal artery biopsy L ) Visual evoked potentials 28. A 57-year-old man is brought to the emergency department 6 hours after the onset of weakness of his right face, arm, and leg. Three days ago, he had an episode of sudden visual loss in his left eye that he describes as "a shade coming down." The episode resolved completely within 10 minutes. He has hypertension and type 2 diabetes mellitus, both poorly controlled with lisinopril and glyburide. Examination shows expressive aphasia and right lower facial droop. There is moderate weakness on the right, worse in the upper extremity than the lower extremity. Deep tendon reflexes are 3+ in the right extremities and 2+ in the left extremities. Babinski's sign is present on the right. Sensory examination shows no abnormalities. For each patient with weakness, select the most appropriate test to establish the cause of the condition. A ) Brain stem auditory evoked potentials B ) Carotid ultrasonography C ) Chromosomal analysis for trinucleotide repeat D ) Electroencephalography E ) Electromyography and nerve conduction studies F ) MRI of the spine G ) Muscle biopsy H ) Repetitive nerve stimulation

I ) Somatosensory evoked potentials J ) SPECT scan K ) Temporal artery biopsy L ) Visual evoked potentials 29. A previously healthy 27-year-old woman is brought to the emergency department because of a 2-day history of weakness of her arms and legs and numbness of her hands and feet and a 4-hour history of mild shortness of breath while supine. The weakness began in her feet and has progressed to involve the hands. She describes a sensation of "electrical shocks" extending from the buttocks to the feet. Today, she tripped and fell several times and was unable to button clothes or hold utensils. Three weeks ago, she had a mild upper gastrointestinal illness that resolved within 5 days. Her respirations are 20/min and shallow. There is mild facial weakness; cranial nerves are otherwise intact. Muscle strength in the upper and lower extremities is 4/5 proximally and 2/5 distally. There is areflexia. Babinski's sign is absent bilaterally. Sensation to vibration is slightly decreased at the fingers and toes. 30. A 72-year-old man comes to the physician with his wife because of chronic abdominal pain and headaches for 4 months. His wife states that he has become more forgetful over the past 6 months. He has a history of gout. He has smoked one pack of cigarettes daily for 50 years and drinks 10 oz of homemade whiskey daily. He takes no medications. His temperature is 36.8 C (98.3 F), blood pressure is 160/98 mm Hg, pulse is 74/min, and respirations are 16/min. Neurologic examination shows mild short-term memory loss and decreased sensation to pinprick in the distal extremities. He has an ataxic gait. There are gouty tophi on the dorsal aspect of the left elbow. Laboratory studies show: Hematocrit 33% Mean corpuscular volume 70 m3 Serum Urea nitrogen (BUN) 17 mg/dL Glucose 90 mg/dL Creatinine 2 mg/dL Uric acid 14 mg/dL Which of the following is the most appropriate next step in management? A ) Measurement of blood lead level B ) Measurement of serum porphobilinogen level C ) CT scan of the abdomen D ) MRI of the brain E ) Carbidopa-levodopa therapy 31. A county health officer investigates an outbreak of illness among persons attending a church

picnic. The illness is characterized by the onset of nausea and vomiting 3 to 4 hours after attending the picnic. All affected persons recover without specific therapy. The investigation implicates egg salad as the vehicle of transmission. This episode is consistent with a foodborne outbreak caused by which of the following? A ) Clostridium perfringens B ) Giardia lamblia C ) Salmonella species D ) Staphylococcus aureus 32. A 62-year-old woman comes to the physician because of generalized weakness for 2 weeks. She has a 20-year history of arthritis of the hands treated with aspirin and acetaminophen. She had two episodes of urinary tract infections 5 and 11 years ago, respectively. Examination shows no abnormalities except for Heberden's nodes on the hands. Laboratory studies show: Erythrocyte sedimentation rate 15 mm/h Serum Na+ 136 mEq/L Cl 100 mEq/L K+ 4.9 mEq/L HCO3 20 mEq/L Urea nitrogen (BUN) 41 mg/dL Creatinine 4 mg/dL Urine Protein 1+ WBC 24/hpf RBC none Bacteria none Squamous epithelial cells occasional Granular casts occasional Renal ultrasonography shows no abnormalities. Which of the following is most likely to have prevented this condition? A ) Periodic PPD skin testing B ) Screening for autoimmune causes of glomerulonephritis

C ) Periodic renal ultrasonography D ) Avoidance of analgesics E ) Suppressive antibiotic therapy for treatment of urinary tract infections 33. A 60-year-old man comes to the physician because of increasing shortness of breath for 1 week; it occurs at rest and is exacerbated by exertion. He has not had chest pain. He has smoked two packs of cigarettes daily for 40 years. Between the ages of 18 and 30 years, he worked in a warehouse with exposed bare insulation; for the past 15 years, he has driven a taxi. Medications include ipratropium bromide and albuterol metered-dose inhalers. His temperature is 37 C (98.6 F), blood pressure is 170/95 mm Hg, pulse is 100/min, and respirations are 24/min. Anterior and posterior diffuse wheezes are heard. Cardiac examination shows no murmurs. There is mild pretibial edema. An x-ray film of the chest shows calcified pleural plaques on the right hemidiaphragm and a 2-cm pleural-based mass. With regard to the findings on the x-ray film, which of the following is most likely to have prevented this patient's condition? A ) Appropriate immunizations B ) Different occupation C ) Different medications D ) Hypertension screening E ) Smoking cessation 34. A previously healthy 67-year-old man is admitted to the hospital because of lethargy, confusion, muscle cramps, and decreased appetite for 7 days. He appears ill. His temperature is 37 C (98.6 F), blood pressure is 120/70 mm Hg, pulse is 98/min, and respirations are 20/min. Breath sounds are diminished at the right lung base. Neurologic examination shows no abnormalities except for lethargy. Serum studies show: Na+ 114 mEq/L K+ 4.3 mEq/L Creatinine 1 mg/dL Thyroid-stimulating hormone 4.1 U/mL An x-ray film of the chest shows a 2-cm nodule in the right lower lobe and mediastinal adenopathy. A biopsy specimen of the nodule is most likely to show which of the following? A ) Adenocarcinoma B ) Clear cell carcinoma C ) Mesothelioma

D ) Small cell carcinoma E ) Squamous cell carcinoma 35. A 62-year-old woman comes to the physician because of bloating and cramping abdominal pain and intermittent diarrhea over the past 5 years. Her symptoms have increased over the past month since she started a new diet that emphasizes yogurt and cottage cheese as low-fat sources of calcium and protein. Vital signs are within normal limits. Abdominal examination shows diffuse tenderness to palpation with no rebound tenderness; there are no masses or organomegaly. Bowel sounds are increased. Test of the stool for occult blood is negative. Which of the following is the best explanation for this patient's diarrhea? A ) Impaired intestinal motility B ) Inflammatory process C ) Malabsorption D ) Secretory process 36. A 52-year-old woman comes to the physician because of a 3-month history of intermittent bloody discharge from the right breast. She does not perform regular monthly breast selfexaminations. She has a 3-year history of major depressive disorder treated with fluoxetine. Examination of the breasts shows no abnormalities. No masses are noted on palpation. Serosanguineous fluid can be expressed from the nipple of the right breast by pressing on the left side of the areola. Which of the following is the most likely diagnosis? A ) Cystosarcoma phyllodes B ) Fat necrosis C ) Fibroadenoma D ) Fibrocystic changes of the breast E ) Galactorrhea F ) Hyperprolactinemia G ) Intraductal papilloma H ) Mastitis I ) Paget's disease of the breast 37. A 6-month-old girl is brought to the physician because of poor feeding and labored breathing for 2 months. She has had recurrent respiratory tract infections since birth. Examination shows a to-

and-fro murmur in the second left intercostal space, a loud S2, bounding peripheral pulses, and a widened pulse pressure. Which of the following is the most likely diagnosis? A ) Atrial septal defect (ostium primum type) B ) Atrial septal defect (ostium secundum type) C ) Atrioventricular canal D ) Coarctation of the aorta E ) Hypoplastic left heart syndrome F ) Patent ductus arteriosus G ) Tetralogy of Fallot H ) Transposition of the great arteries I ) Tricuspid atresia J ) Ventricular septal defect 38. A 70-year-old man comes to the physician because of urinary hesitancy and frequency for 9 months. His temperature is 37.5 C (99.5 F). Examination shows a circumcised penis with no urethral discharge. Testicular examination shows no abnormalities. Rectal examination shows an enlarged rubbery prostate that is nontender to palpation. Urinalysis shows many leukocytes and no erythrocytes. Gram's stain of urine shows gram-negative rods. Which of the following is the most likely cause of this patient's condition? A ) Infection of the epididymis B ) Infection of the prostate C ) Infection of the urethra D ) Neurogenic bladder E ) Outflow obstruction of the bladder 39. A 32-year-old woman comes to the physician because of lethargy and boredom since the birth of her son 5 months ago. She worries about her ability to care for him and has had frequent palpitations. She is unable to fall back asleep after nighttime feedings. She stopped breast-feeding 1 month ago. Her son is healthy, and growth and development are normal for his age. Her blood pressure is 122/80 mm Hg, pulse is 58/min, and respirations are 18/min. Physical examination shows no abnormalities. She remembers one out of three objects after 5 minutes. Her serum cholesterol level is 265 mg/dL. The most appropriate next step in diagnosis is measurement of

which of the following serum levels? A ) Cortisol B ) Estrogen C ) Progesterone D ) Prolactin E ) Thyroid-stimulating hormone 40. The genetic disease institute at a university hospital has developed a rapid screening test for a serious but treatable inherited metabolic disorder. Although this disorder is predominantly found in a particular ethnic group, it is also found sporadically throughout the entire population. The screening test has a sensitivity of 95% and a specificity of 90%. When used in an ethnically prescreened population where the prevalence of this disorder is 30%, the positive predictive value is 80% and the negative predictive value is 96%. The institute proposes to use this screening test on the general population where the prevalence of this disease is 0.1%. Which of the following is the most likely result of this screening program? A ) Negative predictive value decreases B ) Positive predictive value decreases C ) Sensitivity decreases D ) Specificity decreases 41. A 32-year-old woman comes to the physician because of a 1-year history of increasingly severe dull pain in her lower back and buttocks. She also has had morning stiffness of the lower back that lasts for 1 to 2 hours. There is no history of trauma. Her job does not require heavy lifting. Her blood pressure is 110/70 mm Hg, pulse is 68/min and regular, and respirations are 16/min. Cardiopulmonary examination shows no abnormalities. There is tenderness to palpation over the sacroiliac joints bilaterally and decreased flexion and extension of the lumbar spine. An x-ray film of the lumbosacral spine shows sclerosis of the sacroiliac joints. Laboratory studies are most likely to show which of the following findings in this patient? A ) Decreased erythrocyte sedimentation rate B ) Histocompatibility human leukocyte antigen B27 C ) Increased serum antinuclear antibody titer D ) Increased serum carcinoembryonic antigen (CEA) level E ) Positive serum rheumatoid factor

42. A 6-month-old boy is brought to the physician because of respiratory distress for 1 day. He had a persistent dry cough 3 days ago. He has not had a fever or nasal discharge, but he has had chronic watery stools. He is at the 50th percentile for height and 10th percentile for weight. He appears ill and is in respiratory distress. His temperature is 37 C (98.6 F), pulse is 140/min, and respirations are 78/min. Pulse oximetry shows an oxygen saturation of 70% while breathing room air. Examination shows white plaques on the mucous membranes of his mouth and diffuse adenopathy. There are intercostal retractions, and diffuse crackles are heard throughout all lung fields. In addition to oxygen and antibiotic therapy, which of the following is the most appropriate next step in management? A ) Stool culture for bacterial pathogens B ) Urinalysis C ) HIV testing D ) Quantitative measurement of immunoglobulins E ) Nitroblue tetrazolium testing F ) Platelet morphology evaluation G ) Tympanocentesis 43. A 42-year-old man is brought to the physician by his wife because of a 2-month history of staring spells that last 1 to 2 minutes each. During episodes, he also smacks his lips and picks at his shirt collar. Four years ago, he was comatose for 2 weeks after sustaining a head injury in a motorcycle collision; he required 6 months of rehabilitation. He reports that, over the past year, he has had intermittent episodes of smelling burnt rubber that occur approximately every 2 weeks. He hears an intense hissing sound during these episodes. Examination shows no abnormalities. Which of the following is the most likely diagnosis? A ) Absence seizures B ) Complex partial seizures C ) Transient ischemic attack D ) Tourette's disorder E ) Limbic encephalopathy 44. A 47-year-old woman is admitted to the hospital for evaluation of a 2-week history of increased irritability and racing thoughts. Her family reports that she argues with anyone who does not agree with her. She has been talking incessantly at a rapid rate and pacing around the house. She says that, for years, she has been hearing the voice of God telling her that she has been chosen for a special mission. She has barely slept for the past week, waiting to hear further messages from God. She is disheveled and dehydrated. Serum studies show:

Na+ 137 mEq/L Cl 96 mEq/L K+ 3.8 mEq/L HCO3 22 mEq/L Urea nitrogen (BUN) 30 mg/dL Glucose 68 mg/dL Creatinine 1.2 mg/dL Mental status examination shows disorganized thoughts and flight of ideas. She is agitated and has difficulty staying seated. She is too distractible to perform cognitive tests. Toxicology screening is negative. Which of the following is the most likely diagnosis? A ) Acute renal failure B ) Borderline personality disorder C ) Diabetes insipidus D ) Narcissistic personality disorder E ) Schizoaffective disorder F ) Schizoid personality disorder 45. A 20-year-old man comes for a routine health maintenance examination. He has a 15-year history of difficulty relaxing his hands after tightly gripping objects or after shaking hands. His father has cataracts and frontal baldness. Examination shows thin forearms. There is moderate weakness of the hands and difficulty releasing after gripping with his hands. Which of the following is the most likely diagnosis? A ) Amyotrophic lateral sclerosis B ) Cervical spondylosis C ) Multiple sclerosis D ) Myasthenia gravis E ) Myasthenic (Lambert-Eaton) syndrome F ) Myotonic muscular dystrophy G ) Polymyalgia rheumatica H ) Polymyositis

I ) Progressive neuropathic (peroneal) muscular atrophy 46. A 47-year-old man is admitted to the hospital after threatening to harm a radio announcer he believed was broadcasting his thoughts. Over the past 20 years, he has had multiple psychiatric hospitalizations for threatening people who he believed were plotting against him, trying to control his mind, or causing him to hear voices by implanting devices in his head. Past symptoms improved with neuroleptic therapy; after discharge, he discontinued the medication and his symptoms worsened. Which of the following is the most appropriate pharmacotherapy to decrease this patient's risk for future hospitalization? A ) Clozapine B ) Fluphenazine hydrochloride C ) Haloperidol decanoate D ) Risperidone E ) Trifluoperazine hydrochloride ___________________________________________________________________ Answers: FORM1 block 1: 1) e 2) b 3) b 4) d 5) c 6) f 7) d 8) a 9) b 10) h 11) d 12) a 13) g? 14) a 15) f 16) c 17) c 18) f 19)e 20) d 21) b 22) d 23) d 24) e 25) d 26) d 27) a 28) a 29) b 30) d 31) e 32) b 33) b 34) a? 35) b 36) d 37) e 38) e 39) e? 40) d 41) c 42) a 43) c 44) e 45) a 46) d FORM 1 block 2 1) b 2) d 3) d 4) d 5) d 6) e 7) a 8) e 9) g 10) d 11) e 12) d 13) b 14) e 15) c 16) d 17) g 18) a 19)h 20) l 21) b 22) b 23) e 24) e 25) e 26) c 27) d 28) e 29) a 30) a 31) b 32) d 33) c 34) g 35) e 36) e 37) d 38) d 39) d 40) e 41) d 42) d 43) b 44) b? 45) b 46) b Form 1 block 3 1) f 2) c? 3) b 4) c 5) e 6) c 7) a 8) a? 9) a 10) d? 11) b 12) c 13) a 14) e 15) d 16) b 17) b 18) g 19)i?? 20) c 21) c 22) d 23) a 24) c 25) c 26) c 27) a? 28) e 29) a? 30) b 31) e 32) g 33) b 34) b 35) a 36) d 37) f 38) b 39) b? 40) a? 41) d 42) e 43) e 44) d 45) b 46) g? Form 1 block 4 1) H/M 2) e 3) d 4) d 5) d 6) g 7) f 8) p 9) b 10) h 11) h 12) h? 13)i 14) b? 15) b 16) d 17) d 18) e 19)c 20) b 21) a? 22) c 23) e 24) d 25) a 26) e 27) a 28) b 29) e 30) a 31) d 32) d 33) b 34) d 35) c 36) g 37) f 38) e 39) e 40) b 41) b 42) c 43) b 44) e 45) f 46) c More answers: NBME FORM 1 ANSWERS BLOCK 1 1- C 2 -B 3 -B 4- B 5 -C

6 -F 7- D 8 -B 9 -H 10 11-C / B ? 12- A 13- G 14 C 15- F 16-C 17- B 18- F 19- E 20 D 21- B 22-D / C ? 23-B 24-E 25-D 26-D 27-A 28-A 29-B 30- B 31- E / D 32-C 33-C 34-A 35-B 36-D 37-E 38-B 39-E 40-D 41-C 42-A 43-C 44-E 45-A 46-D block 2 1-b 2-d? 3-a 4-d 5-d 6-e 7-f 8-e / next ? c 9-g?? 10-c 11-e 12-d 13-B/ d?? 14-e 15-c 16- D 17- G 18- E 19- H

20- K 21- B 22- B 23- E 24- E 25- E 26- C 27- B 28- E 29- A 30- A 31- A 32- C / D 33- B 34- F 35- E 36- E 37- D 38- D 39- D 40- E 41- D 42- D / E 43- B 44- B 45- B 46- B BLOCK 3 1F 2C 3B 4B 5E 6C 7A 8 C OR E ( if strept pyogens is the same as strept pneumonia then its E) 9A 10 E 11 C 12 C 13 A 14 E 15 D 16.a 17.b?g? 18.g 19.a?c? 20.b?d? 21.c 22.a 23.a 24.c 25.a?b? 26.c 27.a?b? 28.e?g? 29.a 30.b 31 B 32 B 33 E 34 B

35 36 37 38 39 40 41 42 43 44 45 46

A D A B (why such a low bp in this patient,)? E B D C E D B G

BLOCK 4 1-m 2-e 3-e 4-d 5-d 6-g 7-f 8-q 9-a 10-a 11-h 12-?a 13-c 14-?b 15-b 16-?c 17-c 18-e 19-?c 20-b 21-a 22-c 23-e 24-d 25-b 26-e 27-c 28-?b 29-e 30-a 31-d 32-d 33 e 34-d 35-c 36-g 37-f 38-e 39-e 40-b 41-b 42-c 43-e 44-e 45-i 46-a

You might also like